Sie sind auf Seite 1von 69

2

INDEX
01-04-2019 ........................................................................................................................................................... 3

02-04-2019 .......................................................................................................................................................... 6

03-04-2019 ..........................................................................................................................................................8

04-04-2019 ........................................................................................................................................................ 12

05-04-2019 ........................................................................................................................................................ 14

06-04-2019 ........................................................................................................................................................ 17

08-04-2019 ....................................................................................................................................................... 20

09-04-2019 ........................................................................................................................................................ 22

10-04-2019 ......................................................................................................................................................... 25

11-04-2019 .........................................................................................................................................................28

12-04-2019 ......................................................................................................................................................... 31

13-04-2019 ......................................................................................................................................................... 33

15-04-2019 ......................................................................................................................................................... 35

16-04-2019 ......................................................................................................................................................... 37

17-04-2019 ........................................................................................................................................................ 40

19-04-2019 .........................................................................................................................................................42

20-04-2019 ........................................................................................................................................................ 45

22-04-2019......................................................................................................................................................... 47

23-04-2019.........................................................................................................................................................50

24-04-2019 ........................................................................................................................................................ 53

25-04-2019 ......................................................................................................................................................... 55

26-04-2019 ........................................................................................................................................................58

27-04-2019 ........................................................................................................................................................ 60

29-04-2019 ........................................................................................................................................................ 63

30-04-2019 ........................................................................................................................................................66

www.shankariasacademy.com | www.iasparliament.com
3

01-04-2019 4) Consider the following statements with


respect to Turmeric (Curcuma longa)
1) Consider the following statements with
respect to Model Code of Conduct (MCC) 1. Turmeric is a perennial herbaceous
plant of the ginger family.
1. Under MCC, Ministers must not
combine official visits with election 2. India is the world’s largest producer of
work. turmeric and Tamilnadu is the largest
grower of turmeric in India.
2. If any candidate violates MCC, it can be
litigated in any of the high courts but 3. Turmeric with high curcumin content
within the period of operation of MCC. have very high demands in markets.
3. MCC is a part of the Representation of Which of the statement(s) given above is/are
the People Act, 1951. correct?
Which of the statement(s) given above is/are a. 3 only
correct? b. 1 and 2 only
a. 1 only c. 1 and 3 only
b. 1 and 3 only d. 1, 2 and 3
c. 2 and 3 only
d. 1, 2 and 3 5) Consider the following statements with
respect to Dhole (Asiatic Wild Dog)
2) Fridays for Future Movement sometimes 1. It is an apex social carnivore endemic to
seen in the news recently was associated with tropical forests of India.
which of the following? 2. It has been categorized as endangered
a. Climate Change under the IUCN Red list.
b. Nuclear disarmament 3. Dholes are named wild dogs because,
they are genetically similar to dogs.
c. Global level Protest against Terrorism
Which of the statement(s) given above is/are
d. None of the above correct?
a. 2 only
3) Consider the following statements with b. 3 only
respect to the difference between Stalagmites
and Stalactites c. 1 and 2 only
1. Stalagmites hang from the ceiling of a d. 2 and 3 only
cave while Stalactites grow from the
cave floor.
6) Consider the following statements with
2. Most stalactites have pointed tips respect to Pradhan Mantri Jan-Aushadhi
whereas most stalagmites have rounded scheme
or flattened tips.
1. It aims to provide quality drugs at a 50-
Which of the statement(s) given above is/are 90 per cent discount on their branded
correct? counterparts.
a. 1 only 2. Under the Scheme, generic medicines
b. 2 only are separated by a colour code that
differentiate itself from branded
c. Both 1 and 2 medicines.
d. Neither 1 nor 2

www.shankariasacademy.com | www.iasparliament.com
4

Which of the statement(s) given above is/are 1. It is a multilateral development bank


correct? jointly founded by the BRICS countries.
a. 1 only 2. It is headquartered in Manila,
Philippines.
b. 2 only
3. All members of the United Nations
c. Both 1 and 2
could be members of NDB.
d. Neither 1 nor 2
Which of the statement(s) given above is/are
correct?
7) Consider the following statements with a. 1 only
respect to Hump-backed Mahseer
b. 1 and 2 only
1. It is a large freshwater fish found only
c. 1 and 3 only
in the Cauvery river basin.
d. 1, 2 and 3
2. It has been categorized as Least Concern
in the IUCN Red list.
Which of the statement(s) given above is/are 10) Arrange the following hills in order from
correct? South to North
a. 1 only 1. Cardamom Hills
b. 2 only 2. Guru Sikhar
c. Both 1 and 2 3. Kodachadri Hills
d. Neither 1 nor 2 4. Mahendra Giri
Select the correct answer using the codes given
below:
8) Consider the following statements with
respect to Pradhan Mantri Rojgar Protsahan a. 1-4-3-2
Yojana
b. 1-2-3-4
1. It aims to provide conducive
c. 3-1-2-4
environment for entrepreneurs and
incentivize them to promote industrial d. 1-3-4-2
development.
2. All startups are applicable to avail the
benefits granted under the scheme. Answers
3. It comes under the Ministry of Micro, 1. a
Small and Medium Enterprises • Model Code of Conduct (MCC) is not
(MSME). statutory.
Which of the statement(s) given above is/are
• MCC is not enforceable by law.
correct?
2. a
a. 3 only
• Fridays for Future (also called Youth for
b. 1 and 2 only
Climate and Youth Strike 4 Climate) is a
c. 1, 2 and 3 coming together of school students across the
world to tackle the issue of climate change.
d. None
• Since August 2018, there have been
widespread and frequent strikes.
9) Consider the following statements with
respect to New Development Bank (NDB)

www.shankariasacademy.com | www.iasparliament.com
5

• Students have been demanding that • In India Dhole are found in landscapes
governments cut greenhouse gas emissions covering Karnataka portion of Western Ghats.
and frame stronger environmental laws to
fight climate change and secure their future. • IUCN red book lists Dholes as endangered.

3. b • It is included in CITES – Appendix II (2013)


and are legally protected in the countries
• A stalactite is an icicle-shaped formation where they occur.
that hangs from the ceiling of a cave, and
is produced by precipitation of minerals from • The term wild dog is a misnomer as dholes
water dripping through the cave ceiling. Most are genetically distinct from dogs.
stalactites have pointed tips. • They also do not fit into any of the subfamilies
• A stalagmite is an upward-growing mound like foxes or wolves and are classified in a
of mineral deposits that have precipitated from genus of its own called Cuon.
water dripping onto the floor of a cave. Most • They do not possess the strength of big cats,
stalagmites have rounded or flattened but they kill prey larger than themselves by
tips. hunting in packs by biting off chunks of meat
4. c and tearing the animal apart.

• India is the world’s largest producer of • Despite common myths, they are very shy of
turmeric (Curcuma longa), a perennial human presence.
herbaceous plant of the ginger family. • Anthropogenic factors, which fragment and
• The plant’s underground stems or rhizomes alter landscapes seriously affects dholes.
have been used as spice, dye, medicine and 6. a
religious maker since antiquity.
• The central government is favourably
• Tamil Nadu is the third largest grower of considering an amendment to the decades-old
turmeric in the country (behind Telangana Drugs and Cosmetics Rules, to specify a
and Maharashtra), with 132.4 tonnes produced separate colour code for generic medicines,
in 2015-16. from branded medicines.
• The spice’s colour comes mainly • Pradhan Mantri Jan-Aushadhi scheme aims to
from curcumin, a bright yellow phenolic provide quality drugs at a 50-90 per cent
compound that has been in the news for its discount on their branded counterparts.
ostensible potential to fight cancer.
• It is because of this price differential between
• As a result, the demand for turmeric with high generic and branded medicines that the
curcumin content has risen, with government also thinks it is important to have
pharmaceutical companies willing to pay up to a separate colour code.
Rs. 20,000 per quintal for such varieties (up
from Rs. 7,000 earlier). • Since the government has not specified any
features to differentiate generic medicines
• Recently, Erode Turmeric got GI tag in India. from branded, the trade is at times duping the
It has a high curcumin content of around 3.9%. unaware consumer.
• The loamy red and black soil of the area is 7. a
believed to be the reason behind the distinctive
brilliant yellow colour, as well as its • The Humpbacked Mahseer (or) Tor
characteristic sweet taste and aroma, making it remadeviiis is the largest known species of
the preferred choice of commercial curry Mahseer found across the world.
powder manufacturers in India and abroad. • It is found only in the Kaveri river
5. a basin.
• Dhole (commonly known as the Asiatic wild • This fish is also called as tiger of the water
dog) is an apex social carnivore in the tropical found in Pambar, Kabini and Bhavani rivers of
forests of South and South East Asia.

www.shankariasacademy.com | www.iasparliament.com
6

the Kaveri basin in the states Karnataka, 02-04-2019


Kerala and Tamil Nadu.
1) Consider the following statements with
• Recently the fish has been added to the IUCN respect to EMISAT which was launched
Red List as Critically Endangered. recently
• The effects of construction of dams, regulated 1. It is India’s first electronic surveillance
flows, deforestation, drought, pollution and satellite designed and developed by
sediment transport has a great toll on this river ISRO.
water species.
2. It detects electronic signals on ground,
8. d especially hidden enemy radars.
• Pradhan Mantri Rojgar Protsahan Yojana aims Which of the statement(s) given above is/are
to promote/incentivize employment correct?
generation.
a. 1 only
• It comes under the Ministry of Labour and b. 2 only
Employment.
c. Both 1 and 2
• Central Government will be paying the 8.33%
EPS contribution of the employer for the new d. Neither 1 nor 2
employment.
• All industries registered with Employees' 2) Consider the following statements
Provident Fund Organization (EPFO) can
apply for availing benefits under the scheme. 1. La Nina is an abnormal warming of
water in the Equatorial Pacific Ocean.
• It is targeted for employees earning
wages less than Rs. 15,000/- per month. 2. El Nino is the opposite of La Nina, and
refers to the abnormal cooling of the
• The scheme will be in operation for a period of ocean temperatures in the same Pacific
3 years and GOI will continue to pay and all region.
new eligible employees will be covered till
2019-20. Which of the statement(s) given above
is/are incorrect?
9. c
a. 1 only
• New Development Bank is a multilateral
development bank jointly founded by b. 2 only
the BRICS countries. c. Both 1 and 2
• Its focus is to finance infrastructure and d. Neither 1 nor 2
sustainable development in emerging market
and developing countries.
3) Operation P-Hunt sometimes seen in the
• HQ - Shanghai, China.
news recently was associated with which of the
• The first regional office of the NDB will be following?
opened in Johannesburg, South Africa. a. Potential Olympians search operation
• All members of UN could be members of the b. Eliminating Child Pornography
NDB, however the share of the BRICS nations
can never be less than 55% of voting power. c. Prohibiting the spread of Left Wing Extremism
10. d d. Smuggling of threatened species from India

4) Which one of the following best describes


the term Pink Tax?

www.shankariasacademy.com | www.iasparliament.com
7

a. It is the invisible cost that women have to pay b. Assam


for products designed specifically to them
c. Tripura
b. It refers to a special tax levied on products that
d. Andhra Pradesh
contains carcinogen which causes breast
cancer
c. It refers to the tax charged on products that are 9) Consider the following statements with
used exclusively by women respect to National Mineral Development
Corporation (NMDC)
d. Both b & c
1. NMDC is the single largest iron ore
producer in India.
5) Vanuatu is an Island country located in
2. NMDC is under the administrative
which of the following oceans?
control of the Ministry of Steel.
a. North Indian Ocean
Which of the statement(s) given above is/are
b. South Pacific Ocean correct?
c. South Atlantic Ocean a. 1 only
d. West Pacific Ocean b. 2 only
c. Both 1 and 2
6) Project Kautilya sometimes seen in the d. Neither 1 nor 2
news recently was associated with which of the
following?
10) Which of the following comes under
a. Tax evasion
the Eight Core Industries?
b. EMISAT
1. Electricity
c. Terror financing
2. Steel
d. Combat drugs
3. Cement
4. Pesticides
7) Assertion (A): GST exemption on products
Select the correct answer using the codes given
could actually make it costlier
below:
Reason (R): Manufacturer of that product will
a. All except 2
not be able to claim Input tax credit.
b. All except 4
Select the correct answer using the codes given
below: c. All except 1 and 3
a. Both A and R is true and R is the correct d. 1, 2, 3 and 4
explanation of A
b. Both A and R is true but R is not the correct
explanation of A Answers
c. A is true but R is false 1. b
d. A is false but R is true • EMISAT has been developed under DRDO's
Project Kautilya which aims to boost
India's space surveillance capacity was
8) Dhimsa is a popular tribal dance performed launched recently by PSLV-C45.
in which of the following states?
• EMISAT detects electronic signals on ground,
a. Gujarat especially hidden enemy radars.

www.shankariasacademy.com | www.iasparliament.com
8

• EMISAT is modelled after an Israeli spy Intelligence payload for integration on an


satellite called SARAL (Satellite with ARgos indigenous mini satellite.
and ALtika).
• The project is named after the ancient Indian
2. c economist who emphasised the importance of
spying for a king to protect his kingdom.
• El Nino, Spanish for "boy child" (because of
the tendency of the phenomenon to arrive • EMISAT which was launched recently has
around Christmas), is an abnormal warming of been developed under DRDO's Project
water in the Equatorial Pacific Ocean every Kautilya.
three to five years and can last up to 18
7. a
months.
• A manufacturer generally has to pay tax at
• La Nina means “the little girl”, the opposite of
every stage of production, as well on the final
El Nino, and refers to the abnormal cooling of
product.
the ocean temperatures in the same Pacific
region. • The biggest benefit of GST is that now the
3. b government refunds the entire tax it had
collected on inputs or raw materials, provided
• A covert cyber surveillance and infiltration the producer has paid tax on the final product.
operation titled 'Operation P Hunt' was
initiated by the Kerala Police with the help of • As a result, Exemption of the final product
the Interpol in January 2019 to crack down on from GST would mean that manufacturer of a
child pornography. product will not be able to claim tax credit.

4. a • Manufacturers fear that this may make the


final product more expensive.
• Pink tax refers to the invisible cost that
women have to pay for products designed and 8. d
marketed specifically to them, while the • Dhimsa is a tribal dance of the tribes in the
generic or male equivalent of the same eastern part of Andhra Pradesh, like Araku
products are available for less. Valley near Vizag and the tribal regions
• This phenomenon is not limited to first-world of Orissa and Madhya Pradesh.
countries. In India too, women pay pink tax on 9. c
a wide range of products and services
marketed specifically to them. 10. b

5. b 1. Electricity
2. Steel
3. Refinery products
4. Crude oil
5. Coal
6. Cement
7. Natural gas
8. Fertilizers

6. b 03-04-2019
• According to the MoD’s annual report of 2013- 1) Consider the following statements with
14, Kautilya for spaceborne ELINT System respect to Space Debris
involves the development of Electronic

www.shankariasacademy.com | www.iasparliament.com
9

1. It encompasses both natural and 4) Mardani khel is a martial art form prevalent
artificial particles. in which of the following states?
2. ISRO does not have the resources or the a. Assam
capability to track space debris.
b. Manipur
Which of the statement(s) given above is/are
c. Gujarat
correct?
d. Maharashtra
a. 1 only
b. 2 only
5) Consider the following statements with
c. Both 1 and 2
respect to International Space Station (ISS)
d. Neither 1 nor 2
1. It is the world’s only permanent
laboratory in space, orbiting the
geosynchronous orbit.
2) Consider the following statements
2. It is a joint programme collaborated
1. Land and Sea Breezes are local winds
between U.S, Russia, China, France and
caused by the unequal diurnal heating
Europe.
and cooling of adjacent land and water
surfaces. Which of the statement(s) given above is/are
correct?
2. Sea Breeze blows offshore whereas the
land breeze blows onshore. a. 1 only
Which of the statement(s) given above is/are b. 2 only
correct?
c. Both 1 and 2
a. 1 only
d. Neither 1 nor 2
b. 2 only
c. Both 1 and 2
6) Consider the following statements with
d. Neither 1 nor 2 respect to Pradhan Mantri Surakshit Matritva
Abhiyan
1. It aims to provide assured,
3) Which one of the following best describes
comprehensive and quality antenatal
the term Orbital Debris?
care, free of cost, universally to all
a. It refers to all man-made objects in orbit about pregnant women on the 9th of every
the Earth which no longer serve a useful month.
purpose
2. It guarantees a minimum package of
b. It refers to both man-made and natural objects antenatal care services to women in
in orbit about the sun which no longer serve a their 2nd/3rd trimesters of pregnancy
useful purpose at designated government health
facilities.
c. It refers to the debris created by collision of
two or more satellites in and around Low Which of the statement(s) given above is/are
Earth Orbit (LEO) correct?
d. None of the above a. 1 only
b. 2 only
c. Both 1 and 2
d. Neither 1 nor 2

www.shankariasacademy.com | www.iasparliament.com
10

7) Consider the following statements with Which of the pair(s) given above is/are
respect to Financial Action Task Force (FATF) correctly matched?
1. It was set up in 1989 by the G7 a. 3 only
countries.
b. 1 and 2 only
2. It is headquartered in Paris, France.
c. 2 and 3 only
3. India is a member of FATF.
d. 1, 2 and 3
Which of the statement(s) given above is/are
correct?
10) Consider the following statements with
a. 3 only
respect to Jawaharlal Nehru National Solar
b. 1 and 2 only Mission
c. 2 and 3 only 1. It is one of the eight National Missions
under National Action Plan on Climate
d. 1, 2 and 3
Change (NAPCC).
2. The Mission has set the ambitious target
8) Consider the following statements with of deploying 1,00,000 MW of grid
respect to Outer Space Treaty of 1967 connected solar power by 2022.
1. It prohibits countries from placing into 3. It was implemented by the Ministry of
orbit around the Earth any objects New and Renewable Energy.
carrying nuclear weapons or other
Which of the statement(s) given above is/are
weapons of mass destruction.
correct?
2. It also prohibits the stationing of
a. 2 and 3 only
nuclear weapons or other weapons of
mass destruction on celestial bodies, b. 2 only
like the moon, or in outer space.
c. 1 and 3 only
3. India is a signatory to the Outer space
d. 1, 2 and 3
treaty since 1982, but not yet ratified till
now.
Which of the statement(s) given above is/are Answers
correct?
1. c
a. 1 only
• Space debris encompasses both natural
b. 2 only (meteoroid) and artificial (man-made)
c. 1 and 2 only particles.
d. 1, 2 and 3 • ISRO (Indian Space Research Organisation)
does not have the resources or the capability to
track space debris.
9) Consider the following pairs with respect to • It depends on NASA for information on
National Park debris to adjust the flight paths of its launches
1. Pampadum Shola National Park – and existing assets in space.
Karnataka
• NASA and European Space Agency have an
2. Anamudi Shola National Park – elaborate network of radars and optical
Tamilnadu instruments across the globe to monitor and
track the movement of space debris.
3. Mathikettan Shola National Park –
Kerala 2. a

www.shankariasacademy.com | www.iasparliament.com
11

• Land and Sea Breezes are local winds caused • It has 37 members that include all 5
by the unequal diurnal heating and cooling of permanent members of UNSC.
adjacent land and water surfaces; under the
influence of solar radiation by day and • Two regional organisations, the Gulf
radiation to the sky at night, a gradient of Cooperation Council and the European
pressure near the coast is produced. Commission are also its members.

• During the day, the land is warmer than the • Saudi Arabia and Israel are “observer
sea and a breeze, the Sea Breeze, blows countries” (partial membership).
onshore; at night and in the early morning • India became a full member in 2010.
the land is cooler than the sea and the land
breeze blows off shore. 8. c

3. a • The Outer Space Treaty of 1967 prohibits


countries from placing into orbit around the
• Space debris encompasses both natural Earth any objects carrying nuclear weapons or
(meteoroid) and artificial (man-made) other weapons of mass destruction.
particles.
• The treaty also prohibits the stationing of such
• Meteoroids are in orbit about the sun, while weapons on celestial bodies, like the moon, or
most artificial debris is in orbit about the in outer space.
Earth.
• It mandates that outer space, and celestial
• Hence, the latter is more commonly referred to bodies like the Moon, must only be used for
as orbital debris. peaceful purposes.
• Orbital debris is any man-made object in orbit • India is a signatory to this treaty, and
about the Earth which no longer serves a ratified it in 1982.
useful function.
• The Outer Space Treaty prohibits only
• Such debris includes nonfunctional spacecraft, weapons of mass destruction in outer space,
abandoned launch vehicle stages, mission- not ordinary weapons.
related debris and fragmentation debris.
9. a
4. d
List of National Parks in Kerala
• Mardani khel is a martial art indigenous to
Kolhapur region of Maharashtra. 1. Eravikulam National Park

5. d 2. Periyar National Park

• The International Space Station (ISS) is 3. Silent Valley National Park


a collaboration between U.S., Russia, 4. Pampadum Shola National Park
Canada, Europe and Japan.
5. Anamudi Shola National Park
• ISS is located in Low Earth Orbit (LEO).
6. Mathikettan Shola National Park
6. c
10. d
7. d
• Jawaharlal Nehru National Solar
• Financial Action Task Force was set up in 1989 Mission also known as National Solar
by the G7 countries, with headquarters Mission and is also one of the eight National
in Paris. Missions under National Action Plan on
Climate Change (NAPCC).
• It is empowered to curtail financing of UN-
designated terrorist groups. • The Mission has set the ambitious target of
deploying 20,000MW of grid connected solar
• It can publicly sensor countries that are not power by 2022, which was later revised to
abiding by its norms. 1,00,000 MW by 2022.

www.shankariasacademy.com | www.iasparliament.com
12

• The target will principally comprise of 40GW b. Russia – Torpedo


Rooftop and 60GW through large and medium
c. U.S.A – Stealth Submarine
scale grid connected solar power projects.
d. Russia – Surface to surface missile
• There is also 30% capital subsidy for
installation of SPV applications like solar
lighting systems, solar PV power plants and 4) Lord Howe Island is sometimes seen in the
solar pumps under solar off-grid application news for which of the following reasons?
scheme of JNNSM in rural as well as urban
areas throughout India. a. Volcanic eruption
b. Coral bleaching
04-04-2019 c. Ghost fishing

1) Assertion (A): Dark matters in the universe d. 3D printed Island


are hard to detect.
Reason (R): Dark matter easily interacts with 5) Consider the following statements with
the electromagnetic force. respect to Paramparagat Krishi Vikas Yojana
Select the correct answer using the codes given 1. It is a sub-component of Soil Health
below: Management(SHM) scheme under
a. Both A and R is true and R is the correct National Mission of Sustainable
explanation of A Agriculture(NMSA).

b. Both A and R is true but R is not the correct 2. Its main objective is to extend the
explanation of A coverage of irrigation and improving
the water use efficiency.
c. A is true but R is false
Which of the statement(s) given above is/are
d. A is false but R is true correct?
a. 1 only
2) Consider the following statements b. 2 only
1. The Collegium recommends judges for c. Both 1 and 2
both the High Courts and the Supreme
Court. d. Neither 1 nor 2

2. There is no mention of the Collegium


either in the original Constitution of 6) Consider the following statements with
India or in successive amendments. respect to International Sea Bed Authority
Which of the statement(s) given above is/are 1. It is an intergovernmental body
correct? established to regulate all mineral-
a. 1 only related activities in the international
seabed area beyond the limits of
b. 2 only national jurisdiction.
c. Both 1 and 2 2. India is a member of ISA.
d. Neither 1 nor 2 Which of the statement(s) given above is/are
correct?

3) Which of the following is a correct pair with a. 1 only


respect to MH-60 Romeo Seahawks sometimes b. 2 only
seen in the news recently?
c. Both 1 and 2
a. U.S.A – Maritime Helicopter
d. Neither 1 nor 2

www.shankariasacademy.com | www.iasparliament.com
13

b. 2 only
7) Consider the following statements c. Both 1 and 2
1. The union territory of Andaman & d. Neither 1 nor 2
Nicobar Islands comes under the
jurisdiction of Madras High Court.
10) Consider the following statements with
2. The state of Goa has separate High
respect to Deputy Chairman of Rajya Sabha
court.
1. The chairman of Rajya Sabha presides
Which of the statement(s) given above is/are
over the session of election of Deputy
correct?
Chairman.
a. 1 only
2. He/She is elected from amongst the
b. 2 only Rajya Sabha members.
c. Both 1 and 2 3. He/She is directly responsible to the
Rajya Sabha.
d. Neither 1 nor 2
Which of the statement(s) given above is/are
correct?
8) Consider the following statements with
a. 1 only
respect to PENCIL Portal
b. 2 only
1. It serves as a National Digital
Infrastructure for Teachers by enabling c. 1 and 2 only
all teachers across nation to be
d. 1, 2 and 3
equipped with advanced digital
technology.
2. It comes under the Ministry of Human Answers
Resources and Development.
1. c
Which of the statement(s) given above
is/are incorrect? • Unlike normal matter, dark matter does not
interact with the electromagnetic force.
a. 1 only
• This means it does not absorb, reflect or emit
b. 2 only light, making it extremely hard to spot.
c. Both 1 and 2
• In fact, researchers have been able to infer the
d. Neither 1 nor 2 existence of dark matter only from the
gravitational effect it seems to have on visible
matter.
9) Consider the following statements with
• Its gravitational force prevents stars in our
respect to Chandrayaan-2 Mission
Milky Way from flying apart.
1. The purpose of the mission is to collect
2. c
data on the lunar topography,
mineralogy, elemental abundance, 3. a
lunar exposure and signatures of water-
ice. • The U.S administration has recently approved
the sale of 24 multi-role MH-60 Romeo
2. It is the first time India attempts to land Seahawk helicopters to India.
a rover on the moon’s South Pole.
• It is considered the world's most
Which of the statement(s) given above is/are advanced maritime helicopter.
correct?
• The choppers will provide the Indian defence
a. 1 only forces with the capability to perform anti-

www.shankariasacademy.com | www.iasparliament.com
14

surface and anti-submarine warfare 5. Madras HC – Pondicherry and Tamil Nadu


missions.
6. Punjab and Haryana HC – Chandigarh, Punjab
4. b and Haryana
• The Lord Howe Island Group is an outstanding 8. c
example of oceanic islands of volcanic origin
• Platform for Effective Enforcement for No
containing a unique biota of plants and
Child Labour is an electronic platform for no
animals.
child labour developed by the Labour
• The world’s southernmost coral reef off Lord Ministry.
Howe Island has been hit by bleaching this
• The portal creates a robust implementing and
summer.
monitoring mechanism for enforcement of the
• The corals, some 600 km offshore from Sydney legislative provisions of National Child Labour
were affected by elevated temperatures this Policy (NCLP).
summer, despite escaping severe bleaching
9. c
that damaged the Great Barrier Reef in 2016
and 2017. 10. d
• UNESCO records the Lord Howe Island Group
as a World Heritage Site of global natural
significance. 05-04-2019
5. a 1) Which one of the following best explains the
term “Output Gap” in economics?
• It is to promote organic farming and the
a. It refers to the difference between the actual
products will be linked with the market.
output and the maximum potential output of
• It will be implemented in a cluster based an economy
approach and farmers will be funded to meet
b. It indicates the difference between the
the expenditure from farm to market.
investments and revenue of an economy in
• There is no liability for farmers for expenditure terms of GDP
on certification. c. It refers to the ratio between the minimum
6. c output and the maximum output that can be
gained through a transaction
• International Sea Bed Authority is an
intergovernmental body established by the d. None of the above
Law of the Sea Convention to organize,
regulate and control all mineral-related
activities in the international seabed area 2) Consider the following statements with
beyond the limits of national jurisdiction. respect to Political Parties in India
7. d 1. They are obliged to report details of
donations received through Electoral
Common High Courts in India Bonds to the tax authorities in order to
Following are the list of High Courts which have continue to avail IT exemption.
jurisdiction over more than 1 State/UT. 2. They are obliged to disclose the identity
1. Bombay HC - Goa, Dadra and Nager Haveli, of donors of Electoral Bonds to the
Daman and Diu and Maharashtra Election Commission of India (ECI).
2. Calcutta HC- Andaman & Nicobar Which of the statement(s) given above is/are
Islands and West Bengal correct?
3. Guwahati HC- Arunachal Pradesh, Assam, a. 1 only
Mizoram and Nagaland. b. 2 only
4. Kerala HC– Lakshadweep and Kerala c. Both 1 and 2

www.shankariasacademy.com | www.iasparliament.com
15

d. Neither 1 nor 2 6) Consider the following statements with


respect to Section 35AA of Banking Regulation
3) Nagara is a musical instrument belonged to
Act sometimes seen in the news recently
which of the following categories?
1. It grants absolute power to the Reserve
a. Tata Vadya
Bank of India to issue directions to
b. Sushira Vadya banking companies to initiate
insolvency resolution process.
c. Avanaddha Vadya
2. The Supreme Court has recently struck
d. Ghana Vadya down the constitutional validity of
Article 35AA.
4) Which of the following Report(s)/Indices Which of the statement(s) given above is/are
is/are published by World Economic Forum correct?
(WEF)? a. 1 only
1. Global Competitiveness Report b. 2 only
2. Global Gender Gap Index c. Both 1 and 2
3. World Happiness Report d. Neither 1 nor 2
4. Global Education Monitoring Report
Select the correct answer using the codes given 7) Adichanallur sometimes seen in the news
below: recently was one of the earliest ancient sites
a. 1 and 2 only located in which of the following regions?
b. 2 only a. Goa
c. 1, 2 and 3 only b. Kerala
d. 1, 2, 3 and 4 c. Tamil Nadu
d. Puducherry
5) Consider the following statements with
respect to Purchasing Managers' Index 8) Consider the following statements with
1. It is an indicator of economic health for respect to Elections in India
only manufacturing sector. 1. An Observer is an officer of the
2. It will be compiled and released by the government nominated by the Election
Central Statistical Organisation (CSO). Commission of India (ECI).
Which of the statement(s) given above is/are 2. Election manifesto must be consistent
correct? with the letter and spirit of the Model
Code of Conduct.
a. 1 only
Which of the statement(s) given above is/are
b. 2 only correct?
c. Both 1 and 2 a. 1 only
d. Neither 1 nor 2 b. 2 only
c. Both 1 and 2
d. Neither 1 nor 2

www.shankariasacademy.com | www.iasparliament.com
16

9) Porag is a farming related festival • While restricting cash donations to political


celebrated by which of the following parties to less than Rs 2,000 from one donor,
communities? several caveats were provided to maintain
secrecy.
a. Reang
b. Mishing • This includes amending the Income Tax Act,
1961 to relieve political parties from the
c. Lepcha obligation of reporting details of donations
d. Kachari received through Electoral Bonds to the tax
authorities in order to continue to avail IT
exemption.
10) Consider the following statements with • The Representation of the People Act, 1951 was
respect to Rabies also amended to relieve political
1. It is one of the neglected tropical parties from the obligation of maintaining
diseases, caused by virus. the identity of donors who use Electoral Bonds
and disclosing the same to the Election
2. United against Rabies is a collaboration Commission of India.
comprises of WHO to provide a
common strategy to achieve Zero 3. c
human rabies deaths by 2030. • In the Natya Shastra, compiled by Bharat Muni
Which of the statement(s) given above is/are dated 200 B.C. - 200 A.D., musical
correct? instruments have been divided into four main
categories on the basis of how sound is
a. 1 only produced.
b. 2 only 1. The Tata Vadya or Chordophones – Stringed
c. Both 1 and 2 instruments

d. Neither 1 nor 2 2. The Sushira Vadya or Aerophones – Wind


instruments
3. The Avanaddha Vadya or Membranophones –
Answers Percussion instruments
1. a 4. The Ghana Vadya or Idiophones – Solid
instruments which do not require tuning.
• An Output Gap indicates the difference
between the actual output of an economy • Nagara or Kettel drum belongs to the category
and the maximum potential output of an of the Avanaddha
economy expressed as a percentage of Gross Vadyas/Membranophones/Percussion
Domestic Product (GDP). instruments.
• A country's output gap may be either positive • They are further classified under the Sub-
or negative. category of Single-faced drums.
• The Reserve Bank of India (RBI) has recently 4. a
indicated that the output gap of Indian The key reports & indices published by World
economy remained negative and the domestic Economic Forum are
economy was facing headwinds, especially on
the global front. 1. Global Competitiveness Report
2. d 2. Global Enabling trade Report
• Through the Finance Act, 2017, the 3. Global Gender Gap Index
government of India made some changes to 4. Human Capital Index
the statutory scheme of political party funding.
5. Inclusive Development Index

www.shankariasacademy.com | www.iasparliament.com
17

• The World Happiness Report is a measure conduct of election in a constituency or a


of happiness published by the United group of constituencies.
Nations Sustainable Development
• An election manifesto refers to a statement
Solutions Network (UN-SDSN).
issued by a political party fighting an election
• Global Education Monitoring Report is that informs people about the party’s
prepared and published by UNESCO. programmes and policies on a wide range of
issues.
5. d
• The manifesto cannot contain anything that is
• The Purchasing Managers' Index (PMI) is an repugnant to the ideals and principles
indicator of economic health in enshrined in the Constitution and must be
both manufacturing and service sectors. consistent with the letter and spirit of
• The purpose of the PMI is to provide the Model Code of Conduct.
information about current business conditions 9. b
to company decision makers, analysts and
purchasing managers. 10. c

• The PMI is compiled and released monthly by • Rabies is an infectious viral disease.
the U.S based Institute for Supply
• Rabies is one of the neglected tropical
Management (ISM).
diseases that predominantly affect poor and
6. d vulnerable populations who live in remote
rural locations.
• Section 35AA of Banking Regulation
Act empowers the Centre to authorise the • WHO, the World Organisation for Animal
RBI to issue directions to banking companies Health (OIE), the Food and Agriculture
to initiate insolvency resolution process in Organization of the United Nations (FAO) and
respect of a default under provisions of the the Global Alliance for Rabies Control (GARC)
IBC. have established a global “United Against
Rabies” collaboration to provide a common
• Recently, the Supreme Court quashed an RBI strategy to achieve "Zero human rabies deaths
circular issued on February 12, 2018 on by 2030".
stresses assets resolution citing that there
was no authorisation from the Centre
to RBI.
06-04-2019
• However, the Supreme Court order does not
1) Consider the following statements with
curtail the Central Government’s powers to
respect to Flying squirrels
give directions to RBI to initiate recovery
proceedings. 1. They are strictly nocturnal and fly above
the canopy.
7. c
2. They are found in the North East India
• Adichanallur was one of the earliest ancient and the Western Ghats.
sites in Tamil Nadu.
Which of the statement(s) given above is/are
• Carbon dating of samples excavated from the correct?
Adichanallur site in Thoothukudi district by
Beta Analytic Testing Laboratory, USA has a. 1 only
revealed that they belonged to the period b. 2 only
between 905 BC and 696 BC.
c. Both 1 and 2
8. c
d. Neither 1 nor 2
• An Observer, who is an officer of the
government, is nominated by the
Election Commission (EC) to watch the

www.shankariasacademy.com | www.iasparliament.com
18

2) Which of the following countries has c. Odisha


recently launched the world’s first nationwide
d. Tripura
5G mobile networks?
a. Japan
6) LaQshya Initiative comes under which of
b. China
the following ministries?
c. South Korea
a. Ministry of Defence
d. Australia
b. Ministry of Personnel and Public Grievances
c. Ministry of Home Affairs
3) Consider the following statements with
d. Ministry of Health and Family Welfare
respect to Operation Greens
1. Its main objective is to reduce price
volatility in agriculture commodities 7) Consider the following statements with
such as vegetables. respect to Neelkurinji
2. It comes under the Ministry of 1. It is a tropical plant species (shrub)
Agriculture and Farmers Welfare. endemic to the shola forests of the
Western Ghats in South India.
Which of the statement(s) given above is/are
correct? 2. Neelakurinji blooms once in 12 years
and each shrub reproduces once in its
a. 1 only
life time and dies after flowering.
b. 2 only
Which of the statement(s) given above is/are
c. Both 1 and 2 correct?
d. Neither 1 nor 2 a. 1 only
b. 2 only
4) Consider the following with respect c. Both 1 and 2
to Stubble Burning
d. Neither 1 nor 2
1. It is prevalent only in the north and
north-eastern states of India.
8) Consider the following statements with
2. The smoke from stubble contains
respect to Mahatma Gandhi National Rural
carbon monoxide, which adversely
Employment Guarantee Act (MGNREGA)
affect human health and environment.
1. Under the Scheme, Social audit has to
Which of the statement(s) given above is/are
be done by the gram sabha atleast once
correct?
in every 6 months.
a. 1 only
2. Under the Scheme, at least one-third
b. 2 only beneficiaries shall be women.
c. Both 1 and 2 3. It comes under the Ministry of Rural
Development.
d. Neither 1 nor 2
Which of the statement(s) given above is/are
correct?
5) Konyak Tribes live mainly in which of the
a. 1 only
following regions?
b. 1 and 2 only
a. Nagaland
c. 2 and 3 only
b. Assam
d. 1, 2 and 3

www.shankariasacademy.com | www.iasparliament.com
19

9) Consider the following statements with 2. c


respect to National Investment Infrastructure
3. a
Fund
1. It is an Indian-government backed • Operation Greens was launched by
entity established to provide long-term the Ministry of Food Processing
capital to the country’s infrastructure Industries on the lines of Operation Flood.
sector. • The idea behind Operation Greens is to double
2. It is being considered as an Alternative the income of farmers by end of 2022.
Investment Fund (AIF) under SEBI • It aims to promote farmer producers
regulation. organizations, Agri-logistics, processing
Which of the statement(s) given above is/are facilities and professional management.
correct? • It is essentially a price fixation scheme that
a. 1 only aims to ensure farmers are given the right
price for their produce.
b. 2 only
• The main objective of the project is to reduce
c. Both 1 and 2 price volatility in agriculture commodities such
d. Neither 1 nor 2 as vegetables.
• Initially, Government has decided to start
focusing on three basic vegetables namely
10) Which of the following countries has tomatoes, onions and potatoes.
recently granted its highest civilian award “The
Order of Zayed” to Indian Prime Minister? • NAFED will be the Nodal Agency to implement
price stabilisation measures.
a. United Arab Emirates
4. b
b. Indonesia
• In Kuttanad, the rice bowl of Kerala, farmers
c. Qatar
are burning fields to destroy stubble, to check
d. Palestine the germination of weedy rice and prevent
diseases, as part of preparing their fields for
the next crop season.
Answers • They believe setting the fields on fire will
1. c improve the soil fertility.
• Flying squirrels are strictly nocturnal and 5. a
fly above the canopy. • Konyak is one of the 16 Naga tribes and
• Flying squirrels do not actually ‘fly’; their people of this community live mainly in
forelimbs and hind limbs are attached together the Mon district of Nagaland.
with a fold of skin, a membrane. 6. d
• When they jump above the canopy, the • LaQshya Initiative – Labour Room Quality
membrane unfolds and it seems that the Improvement Initiative’s objective is to reduce
squirrel is in flight. preventable maternal and new-born mortality,
• Flying squirrels make their homes in tree morbidity and stillbirths by improving the
holes, if flying squirrels are seen making nests quality of care provided in the labour room.
in tree branches, it can be understood that the • It will be implemented in Government Medical
forest is disturbed. Colleges besides District Hospitals, and Sub-
• Flying squirrels are found in India’s two District Hospitals and Community Health
biggest diversity hotspots: the North Centres.
East (Mostly in Arunachal Pradesh) and
the Western Ghats (Mostly in Kerala)

www.shankariasacademy.com | www.iasparliament.com
20

• The initiative plans to conduct quality c. Endangered species in Arctic Region


certification of labour rooms and also
d. Multi-drug resistant Fungi
incentivize facilities achieving the targets
outlined.
7. b 3) Consider the following statements with
respect to Bicameralism in India
• Neelkurinji is a tropical plant species (shrub)
found in Asia and Australia. 1. Bicameralism was first introduced in
India through the Government of India
• In India, it is found in the shola forests of the Act, 1909.
Western Ghats in South India.
2. Parliament can create or abolish a
• It belongs to the genus Strobilanthes which has legislative council, if the legislative
around 250 species. assembly of the concerned state passes a
• Blooming periods of different species of resolution to that effect by a simple
Kurinji differ from each other. majority.
Which of the statement(s) given above is/are
• Neelakurinji blooms once in 12 years.
correct?
• Each shrub reproduces once in its life time and
a. 1 only
dies after flowering.
b. 2 only
8. d
c. Both 1 and 2
9. c
d. Neither 1 nor 2
10. a

4) Match the following


08-04-2019
1. Hope Mars Mission – a. China
1) Consider the following statements with
respect to Battle of Kangla Tongbi 2. Mars 2020 – b. USA
1. It was fought between the allied forces 3. Mars Global Remote Sensing Orbiter
and Japan during the Second World and Small Rover – c.UAE
War. Select the correct answer using the codes given
2. At the end of this battle, Japanese forces below:
capture the British territory of Imphal. a. 1-a; 2-b; 3-c
Which of the statement(s) given above is/are b. 1-c; 2-b; 3-a
correct?
c. 1-c; 2-a; 3-b
a. 1 only
d. 1-b; 2-a; 3-c
b. 2 only
c. Both 1 and 2
5) Which of the following countries are the
d. Neither 1 nor 2 members of the Group of Seven (G-
7) countries?
2) Candida Auris sometimes seen in the news 1. Canada
recently refers to? 2. France
a. Ionisation of electrified particles in Ionosphere 3. China
b. Phenomenon of bending of light rays by 4. Italy
gravity
5. Russia

www.shankariasacademy.com | www.iasparliament.com
21

Select the correct answer using the codes given c. If a candidate has secured precisely or less
below: than one-sixth of the total no of valid votes cast
by all the candidates at the election
a. All except 1, 4 and 5
d. If a candidate is elected and polled more than
b. All except 3 and 5
one-sixth of the total number of valid votes
c. All except 1 and 4 polled by all the candidates at the election
d. 1, 2, 3, 4 and 5
9) Dandeli wildlife sanctuary is located in
which of the following states?
6) World Health Statistics Overview 2019 is a
report published recently by which of the a. Madhya Pradesh
following agencies?
b. Uttarakhand
a. World Health Organisation
c. Chhattisgarh
b. UNICEF
d. Karnataka
c. The Global Health Network
d. Medecins Sans Frontieres
10) Consider the following pairs
Places in News – Country
7) Consider the following statements with
1. Tripoli – Libya
respect to Pradhan Mantri Ujjwala Yojana
(PMUY) 2. Kigali – Kyrgyzstan
1. It aims to provide 5 crore free LPG 3. Karbi Anglong – Turkey
connections to Women from BPL
Which of the pair(s) given above is/are
Households.
correctly matched?
2. LPG connection is provided in the name
a. 1 only
of the adult women of the family.
b. 1 and 3 only
3. It was launched by the Ministry of
Health and Family Welfare. c. 2 and 3 only
Which of the statement(s) given above is/are d. None
correct?
a. 2 only
Answers
b. 1 and 2 only
1. a
c. 1, 2 and 3
• The Battle of Kangla tongbi also known as
d. None Battle of Lion Box is considered one of the
fiercest battles of World War II.
8) In which of the below scenarios the • The battle was fought between the Axis
candidate forfeits his/her deposit as per the (Japanese) Force and Allied Force on the night
provisions under Representation of the People of 6/7 April 1944.
Act, 1951?
• Japanese forces had planned a three pronged
a. If a candidate is elected but secured less than offensive to capture Imphal and the
one-sixth of the total number of valid votes surrounding areas.
polled by all the candidates at the election
• But, a detachment of the 221 Advance
b. If a candidate is not elected but polled more Ordnance Depot (AOD) put a stiff resistance
than one-sixth of the total no of valid votes cast stalling the advance of Japanese forces and
by all the candidates at the election forced the Japanese to withdraw.

www.shankariasacademy.com | www.iasparliament.com
22

2. d • G7 is a group consisting of Canada, France,


Germany, Italy, Japan, the UK, and the US.
• Candida Auris is a fungus that, when it
gets into the bloodstream, can cause • EU is also represented within the G7.
dangerous infections that can be life-
threatening. 6. a

• It is often resistant to major antifungal • Women outlive men everywhere, the World
drugs that are typically used to treat such Health Organization (WHO) said recently
infections. while releasing its World Health Statistics
Overview 2019.
• Once the germ is present, it is hard to eradicate
from a facility. Some hospitals have had to • Whether it’s homicide, road accidents, suicide,
bring in special cleaning equipment and even and cardiovascular disease— time and time
rip out floor and ceiling tiles to get rid of it. again, men are doing worse than women.

3. d 7. a

• Parliament can abolish a legislative council • The PMUY was launched by the Prime
(where it already exists) or create it (where it Minister Narendra Modi in 2016 with
does not exist), if the legislative assembly of an initial target of 5 crore LPG
the concerned state passes a resolution to that connections, which was later revised upward
effect. to 8 crore connections.

• Such a specific resolution must be passed by • It was launched by the Ministry of


the state assembly by a special majority, Petroleum and Natural Gas.
that is, a majority of the total membership of 8. c
the assembly and a majority of not less than
two-thirds of the members of the assembly 9. d
present and voting. 10. a
• This Act of Parliament is not to be deemed as 1. Tripoli – Libya
an amendment of the Constitution for the
purposes of Article 368 and is passed like an 2. Kigali – Rwanda
ordinary piece of legislation (i.e. by simple 3. Karbi Anglong – India
majority).
• Government of India Act of 1919 introduced,
for the first time, bicameralism and direct 09-04-2019
elections in the country. 1) Consider the following statements with
4. b respect to National Institutional Ranking
Framework (NIRF)
• Hope Mars Mission is a space exploration
probe mission built by the UAE and set for 1. It ranks higher educational institutions
launch in 2020. across the country on both the category-
specific and domain-specific rankings.
• Mars 2020 is NASA’s Mars Exploration
Programme with a planned launch on July 17, 2. It was launched by the Ministry of
2020 and expected to reach the planet on Feb Human Resource and Development.
2021. Which of the statement(s) given above
• Mars Global Remote Sensing Orbiter is/are incorrect?
and Small Rover (HX-1) is a planned a. 1 only
mission by China is expected to be launched
by 2020. b. 2 only

5. b c. Both 1 and 2
d. Neither 1 nor 2

www.shankariasacademy.com | www.iasparliament.com
23

d. Neither 1 nor 2
2) Which of the following is the objective
of Event Horizon Telescope Project sometimes
5) Consider the following statements with
seen in the news recently?
respect to Hunter Committee of Inquiry, 1919
a. To measure upwelling infrared radiation of the
1. It was constituted to investigate the
Earth
Jallianwala Bagh massacre.
b. To capture the first image of a black hole
2. There were no Indian members in the
c. To settle essential questions in the areas of Committee.
dark energy
Which of the statement(s) given above is/are
d. To study the suitability of life in Jupiter’s correct?
Moon Europa
a. 1 only
b. 2 only
3) How is the International Court of Justice
c. Both 1 and 2
(ICJ) different from the International Criminal
Court (ICC)? d. Neither 1 nor 2
1. All the members of UN automatically
become members of ICC whereas
nations must individually sign the Rome 6) Domkhar Rock Art Sanctuary sometimes
statute to become members of ICJ. seen in the news is located in which of the
following states?
2. ICC is a criminal Court whereas the ICJ
is not a Criminal Court. a. Jammu and Kashmir

Which of the statement(s) given above is/are b. Haryana


correct? c. Rajasthan
a. 1 only d. Tamilnadu
b. 2 only
c. Both 1 and 2 7) Consider the following statements with
d. Neither 1 nor 2 respect to Pradhan Mantri Vaya Vandana
Yojana
1. The scheme is meant exclusively for the
4) Consider the following statements with senior citizens aged 60 years or above.
respect to Atal Ranking of Institutions on
Innovation Achievements (ARIIA) 2. Life Insurance Corporation is the
implementing agency for this scheme.
1. The ranking is done on the basis of
teaching, learning and resources, 3. It was launched by the Ministry of Social
research and professional practices, Justice and empowerment.
graduation outcomes, outreach and Which of the statement(s) given above is/are
inclusivity, and perception. correct?
2. It is an initiative of Ministry of Human a. 1 only
Resource Development (MHRD).
b. 1 and 2 only
Which of the statement(s) given above is/are
correct? c. 2 and 3 only

a. 1 only d. 1, 2 and 3

b. 2 only
c. Both 1 and 2 8) Consider the following statements

www.shankariasacademy.com | www.iasparliament.com
24

1. Talcher city is India’s largest coalfield • The Event Horizon Telescope is an


located in Jharkhand. international collaboration aiming to capture
the first image of a black hole by creating a
2. Brahmani is the second largest river in
virtual Earth-sized telescope.
Odisha.
3. b
Which of the statement(s) given above is/are
correct? ICJ
a. 1 only • Principal Organ of UN.
b. 2 only • All the member states of UN automatically
c. Both 1 and 2 become members of ICJ

d. Neither 1 nor 2 • Not a criminal court and therefore, it does not


have a prosecutor able to initiate proceedings
• Settles disputes between member states, with
9) Consider the following pairs: their consent, on issues of sovereignty, trade,
Places in News – Country natural resources, treaty violations, treaty
interpretation, and etc.
1. Komodo Island – Indonesia
ICC
2. Neil Island – Australia
• Set up under the Rome Statute in 2002, not
3. Rodrigues – Mauritius governed by UN
Which of the pair(s) given above is/are • nations must individually become members by
correctly matched? signing Rome Statute
a. 2 only
• Criminal court
b. 1 and 3 only
• Tries individuals for genocide, crimes against
c. 2 and 3 only humanity, war crimes, and crimes of
aggression.
d. 1, 2 and 3
4. b

10) Dhol, Dholak in music refers to which of • Atal Ranking of Institutions on Innovation
the following in India? Achievements (ARIIA) is an initiative of
Ministry of Human Resource Development
a. Hindustani Ragas (MHRD).
b. String Instruments • It aims to systematically rank all major higher
c. Notation systems educational institutions and universities in
India on indicators related to “Innovation and
d. Drum Instruments Entrepreneurship Development” amongst
students and faculties.
Major Indicators are:
Answers
1. Budget & Funding Support.
1. d
2. Infrastructure & Facilities.
• NIRF ranks higher education based on 9
categories: Overall Ranking (1), Category- 3. Awareness, Promotions & support for Idea
Specific Rankings (2 – Universities and Generation & Innovation.
Colleges) and Domain-Specific Rankings (6- 4. Promotion & Support for Entrepreneurship
Engineering, Management, Law, Pharmacy, Development.
Architecture and Medical).
5. Innovative Learning Methods & Courses.
2. b

www.shankariasacademy.com | www.iasparliament.com
25

6. Intellectual Property Generation, Technology • Due to extensive underground coal mining


Transfer & Commercialization. carried out in this area, surface subsidence is
frequent in this area.
7. Innovation in Governance of the Institution.
• The dangers to Talcher are many. If a
• NIRF ranking is done on the basis of
moderate earthquake shakes the area, a large
teaching, learning and resources, research and
part of the township may cave in.
professional practices, graduation outcomes,
outreach and inclusivity, and perception. • If any water channel of the Brahmani River
5. a (Odisha’s second largest River), which
flows a few hundred metres away, finds its way
• The Secretary of State for India, Edwin in to the mining void, it could prove disastrous.
Montagu, ordered that a committee of inquiry
9. b
be formed to investigate the Jallianwala Bagh
massacre. • Neil Island of Andaman and Nicobar Island
was recently renamed as Shaheed Dweep
• On October 14, 1919, the Government of India
Island.
announced the formation of the Disorders
Inquiry Committee, which came to be more • Indonesia has decided to shut Komodo
widely and variously known as the Hunter Island to tourists for atleast a year after it
Committee/Commission after the name of found that the endangered fork-tongued
chairman, Lord William Hunter. komodo dragon was being smuggled out in
large numbers.
• The purpose of the commission was to
“investigate the recent disturbances in • The island of Rodrigues, Mauritius in the
Bombay, Delhi and Punjab, about their causes, Indian Ocean has been battered by Tropical
and the measures taken to cope with them”. Cyclone Joaninha recently.
• There were three Indians among the 10. d
members of the committee.
6. a
10-04-2019
• The sanctuary is located at Domkhar village in
Leh, Jammu and Kashmir next to the banks of 1) Consider the following statements with
Indus River. respect to FAME India II Scheme

• It has 500-odd petroglyphs. 1. It is the expanded version of the FAME


India I scheme which was launched by
7. b the Ministry of Road Transport and
Highways.
• Pradhan Mantri Vaya Vandana Yojana is a
pension scheme announced exclusively for the 2. It aims to encourage faster adoption of
senior citizens aged 60 years or above. electric and hybrid vehicles by way of
offering upfront incentive on purchase
• It was launched by the Ministry of Finance.
of electric vehicles.
• The investment limit is Rs.15 lakh/senior Which of the statement(s) given above is/are
citizen and provides an assured return of 8% correct?
p.a. for 10 years.
a. 1 only
• It is exempted from Service Tax/ GST and LIC
is the implementing agency. b. 2 only
8. b c. Both 1 and 2

• Talcher city of Odisha is India’s largest d. Neither 1 nor 2


coalfield.

www.shankariasacademy.com | www.iasparliament.com
26

2) Arrange the following in descending order 5) Consider the following statements with
based on its gold reserves respect to UN-Industrial Development
Organization (UNIDO)
1. India
1. It is one of the specialised agencies of
2. China
U.N.
3. U.S.A
2. It promotes industrial development for
4. International Monetary Fund poverty reduction, inclusive
globalization and environmental
Select the correct answer using the codes given sustainability.
below:
Which of the statement(s) given above is/are
a. 3-4-2-1 correct?
b. 3-2-4-1 a. 1 only
c. 1-3-2-4 b. 2 only
d. 1-4-3-2 c. Both 1 and 2
d. Neither 1 nor 2
3) Consider the following pairs
Reports – Published by 6) Sampoorna Bima Gram Yojana comes
1. World Economic Outlook – World under which of the following ministries?
Bank a. Ministry of Finance
2. Global Economic Prospects – b. Ministry of Rural Development
International Monetary Fund
c. Ministry of Health and Family Welfare
3. World Happiness Report –
UNESCO d. Ministry of Communications
Which of the statement(s) given above is/are
correct?
7) Which of the following are West flowing
a. 3 only rivers?
b. 1 and 2 only 1. Mahi
c. 1, 2 and 3 2. Zuari
d. None 3. Mandovi
Select the correct answer using the codes given
below:
4) Project Kuiper sometimes seen in the news
recently is an initiative of which of the a. 3 only
following?
b. 1 and 3 only
a. Google
c. 2 and 3 only
b. Microsoft
d. 1, 2 and 3
c. Facebook
d. Amazon
8) Consider the following statements with
respect to Organization for the Prohibition of
Chemical Weapons (OPCW)
1. It is an independent, autonomous
international organisation with a
working relationship with the UN.

www.shankariasacademy.com | www.iasparliament.com
27

2. It is the implementing body of the (FAME) scheme will come into force from
Chemical Weapons Convention (CWC), April 1, 2019.
which entered into force in 1997.
• The scheme will be in effect for a period of
3. It was awarded Nobel Peace Prize in three years at a proposed budget of Rs 10,000
2013. crore.
Which of the statement(s) given above is/are 2. a
correct?
a. 2 only
b. 1 and 2 only
c. 1 and 3 only
d. 1, 2 and 3

9) Consider the following pairs


1. Kaiga – Karnataka
2. Kakrapar – Rajasthan
3. Narora – Gujarat
Which of the pair(s) given above is/are
correctly matched?
a. 1 only
b. 1 and 2 only
c. 1 and 3 only
d. 2 and 3 only 3.d
• World Economic Outlook – International
Monetary Fund
10) Achanakmar Wildlife Sanctuary is located
in which of the following states? • Global Economic Prospects – World Bank
a. Kerala • World Happiness Report – Sustainable
b. Madhya Pradesh Development Solutions Network

c. Chhattisgarh 4. d

d. Jharkhand • In a bid to provide Internet to the "unserved


and underserved communities around the
world", Amazon is working to launch a
constellation of 3,236 satellites into low-Earth
Answers orbit.
1. b
• Project 'Kuiper' will provide low-latency,
• FAME India II Scheme is the expanded version high-speed broadband connectivity to over 95
of the present scheme titled FAME India I per cent of the earth's total population.
which was launched in April 2015 by 5. c
the Ministry of Heavy Industries and
Public Enterprises. • UN-Industrial Development Organization
(UNIDO) is the specialized agency that
• The second phase of the Faster Adoption and promotes industrial development for poverty
Manufacturing of (hybrid) Electric vehicles

www.shankariasacademy.com | www.iasparliament.com
28

reduction, inclusive globalization and 11-04-2019


environmental sustainability.
1) Operation Sahayata - 2019 sometimes seen
• The mission of UNIDO is to promote and in the news recently was associated with which
accelerate inclusive and sustainable industrial of the following?
development (ISID) in Member States.
a. India-Japan Navy
• UNIDO is perfectly positioned to assist
developing countries and countries with b. Coral Rehabilitation
economy in transition to shift from HFCs to c. Idai Cyclone
ozone- and climate-friendly alternatives and to
improve energy efficiency. d. Nutritional Enrichment

6. d
2) Consider the following statements with
• Sampoorna Bima Gram Yojana aims to give
respect to Press Council of India (PCI)
rural people affordable life insurances services.
1. It is a statutory, quasi-judicial body.
• Under the scheme, at least one village (having
a minimum of 100 households) will be 2. It acts as the watchdog of the press.
identified in each districts and provide with a
3. Decisions of the PCI are final and
minimum of one Rural Postal Life Insurance
cannot be appealed before a court of
policy for each households.
law.
• Coverage of all households in the identified
Which of the statement(s) given above is/are
village is the primary objective of this scheme.
correct?
• All villages under the Saansad Adarsh Gram a. 2 only
Yojana will be brought under its ambit.
b. 1 and 2 only
7. d
c. 1 and 3 only
8. d
d. 1, 2 and 3
• Organization for the Prohibition of
Chemical Weapons (OPCW) is an
independent, autonomous international 3) Consider the following statements with
organisation with a working relationship with respect to Black Hole
the UN.
1. It is a region in space where the pulling
• It is the implementing body of the Chemical force of gravity is so strong that even
Weapons Convention (CWC), which entered light can’t escape.
into force in 1997.
2. Event Horizon refers to a point of no
• It has 193 Member States working together to return, beyond which it is impossible to
achieve a world free of chemical weapons. HQ escape the gravitational effects of the
- The Hague, Netherlands black hole.
• It was awarded Nobel Peace Prize in 2013. 3. It helps astronauts to reduce the
9. a distance for long journeys across the
universe.
• Kaiga – Karnataka
Which of the statement(s) given above
• Kakrapar – Gujarat is/are incorrect?
• Narora – Uttar Pradesh a. 3 only
10. c b. 1 and 2 only
c. 1 and 3 only
d. 1, 2 and 3

www.shankariasacademy.com | www.iasparliament.com
29

d. 1, 2 and 3
4) Menhir, Cairn, Dolmen and Cist sometimes
seen in the news refers to which of the
8) Consider the following statements with
following?
respect to Pradhan Mantri Khanij Kshetra
a. Age old stones Kalyan Yojana
b. Ranks used in Guptan era 1. It was launched to provide for the
welfare of areas and people affected by
c. Books written by Shivaji
mining related operations.
d. Archaeological Burial sites
2. It was launched by the Ministry of Rural
Development.
5) Astronauts experience space sickness Which of the statement(s) given above is/are
without Earth’s gravity. This condition is called correct?
________
a. 1 only
a. Space vertigo
b. 2 only
b. Space adaptation syndrome
c. Both 1 and 2
c. Microgravity sickness
d. Neither 1 nor 2
d. None of the above

9) Sinharaja Forest Reserve is located in which


6) Kannanur Reserve Forest is located in of the following countries?
which of the following states?
a. Maldives
a. Kerala
b. Nepal
b. Karnataka
c. Bhutan
c. Telangana
d. Srilanka
d. Tamilnadu

10) Match the following:


7) Consider the following statements with
List I List II
respect to International Labour Organisation
(ILO) 1. Peshwa – a. Gwalior
1. It is an agency of the United Nations 2. Gaekwad – b. Nagpur
dedicated to improving labour
3. Holkars – c. Indore
conditions and living standards
globally. 4. Bhonsle – d. Baroda
2. It was established in 1919 by the Treaty 5. Sindhias – e. Poona
of Versailles as an affiliated agency of
the League of Nations. Select the correct answer using the codes given
below:
3. ILO was awarded the Nobel Prize for
Peace in 2015. a. 1-a; 2-b; 3-c; 4-d; 5-e

Which of the statement(s) given above is/are b. 1-d; 2-c; 3-b; 4-a; 5-e
correct? c. 1-e; 2-d; 3-c; 4-b; 5-a
a. 1 only d. 1-e; 2-c; 3-d; 4-b; 5-a
b. 1 and 2 only
c. 2 and 3 only

www.shankariasacademy.com | www.iasparliament.com
30

Answers 4. a
1. c • The Madras High Court bench has recently
directed the forest department personnel and
• Operation Sahayata 2019: India’s other government officials to protect the
Humanitarian Assistance and Disaster Relief derooted age-old stones in the Kannanur
operation to Mozambique after the landfall reserve forest in Pudukottai district of
of Idai Cyclone. Tamilnadu that were kept on the roadside.
2. b • It is home to rare stones like Menhir, Cairn,
• The Press Council of India (PCI) was Dolmen and Cist.
established under the PCI Act of 1978 for the 5. b
purpose of preserving the freedom of the press
and of maintaining and improving the • Gravity is the force that pulls us down to the
standards of newspapers and news agencies in ground.
India.
• The low gravity in space allows astronauts to
• The decisions of the council are final and float around, which looks very fun, but it can
cannot be questioned in any court of help cause space sickness.
law except by way of writ under
• Space sickness is nausea and disorientation
relevant article of the constitution.
felt by many astronauts.
3. a
• NASA uses the term “Space Adaptation
• A black hole is a region of space from which Syndrome” instead of space sickness.
nothing, not even light, can escape.
• It more closely describes the problem because
• Despite the name, they are not empty but it is an issue of the astronaut struggling to
instead consist of a huge amount of matter adapt to weightlessness in space.
packed densely into a small area, giving it an
• The cause of space sickness is still not fully
immense gravitational pull.
understood.
• There is a region of space beyond the black
6. d
hole called the Event Horizon.
• The Madras High Court bench has recently
• This is a "point of no return", beyond which it
directed the forest department personnel and
is impossible to escape the gravitational effects
other government officials to protect
of the black hole.
the derooted age-old stones in the
• Anything that comes within the event horizon Kannanur reserve forest in
will be consumed forever. Because no light can Pudukottai district of Tamilnadu that
escape from it, a black hole is invisible. were kept on the roadside.
• However, advanced space telescopes can • It is home to rare stones like menhir, cairn,
identify black holes by observing the behaviour dolmen and cist.
of material and stars that are very close to 7. b
black holes.
• The International Labour Organization (ILO)
• This hot disk of material encircling a black is an agency of the United Nations dedicated to
hole, as NASA explained, shines bright. improving labour conditions and living
• And against this disk, a black hole appears to standards globally.
cast a shadow. • The ILO was established in 1919 by the Treaty
• This is how the photograph of the black hole of Versailles as an affiliated agency of the
was achieved. League of Nations.

• A wormhole is a theoretical passage through • The main aims of the ILO are to promote
space-time that could create shortcuts for long rights at work, encourage decent employment
journeys across the universe.

www.shankariasacademy.com | www.iasparliament.com
31

opportunities, enhance social protection and 3) Select the correct pair with respect
strengthen dialogue on work-related issues. to Beresheet sometimes seen in the news
recently
• In recognition of its activities, the ILO was
awarded the Nobel Prize for Peace in 1969. a. Israel – Moon
8. a b. U.S.A – Saturn
• It was launched by the Ministry of Mines c. Israel – Communication satellite
9. d d. Japan – Asteroid Ryugu
10. c
1. Peshwa – Poona 4) Consider the following statements with
respect to solar e-waste
2. Gaekwad – Baroda
1. Solar e-waste contains potentially
3. Holkars – Indore hazardous elements like chromium,
4. Bhonsle – Nagpur silicon tetrachloride, cadmium, and
uranium.
5. Sindhias – Gwalior
2. Currently, India’s e-waste rules have no
laws mandating solar cell
12-04-2019 manufacturers to recycle or dispose
waste from this sector.
1) Bold Kurukshetra 2019 sometimes seen in
Which of the statement(s) given above is/are
the news recently is associated with which of
correct?
the following?
a. 1 only
a. India – Singapore
b. 2 only
b. Domestic Olympics
c. Both 1 and 2
c. NITI Aayog
d. Neither 1 nor 2
d. Women Empowerment program

5) Deori tribes live in which of the following


2) Consider the following statements
states?
1. The Election Commission of India
a. Goa
imposes limits on campaign
expenditure incurred by a candidate, b. Assam
but not by a political party.
c. Maharashtra
2. The Election Commission of India can
allot animals as symbols for political d. Odisha
parties.
Which of the statement(s) given above is/are 6) Consider the following statements with
correct? respect to Fall Army Worm
a. 1 only 1. It is an insect that is native to tropical
b. 2 only and subtropical regions of the Americas.

c. Both 1 and 2 2. It has been spread mostly in southern


states and has not yet spread to the
d. Neither 1 nor 2 north-western India.
3. It cannot tolerate extreme temperatures
or excess rainfall.

www.shankariasacademy.com | www.iasparliament.com
32

Which of the statement(s) given above is/are Which of the pair(s) given above is/are
correct? correctly matched?
a. 3 only a. 1 only
b. 1 and 2 only b. 1 and 2 only
c. 1, 2 and 3 c. 2 and 3 only
d. None d. None
7) Match the following 10) Rajaji Tiger Reseve is located in which of
the following States?
Policies – Proponents
a. Kerala
1. Forward Policy – a. Lytton
b. Karnataka
2. Policy of Masterly Inactivity – b.
Auckland c. Uttarakhand
3. Policy of Proud Reserve – c. John d. Tamilnadu
Lawrence
Select the correct answer using the codes given
below: Answers
a. 1-a; 2-c; 3-b 1. a

b. 1-b; 2-a; 3-c • The 12th edition of joint military exercise


between India and Singapore, Bold
c. 1-a; 2-b; 3-c Kurukshetra 2019, culminated on 11 April
d. 1-b; 2-c; 3-a 2019 after an impressive closing ceremony
held at Babina Military Station, Uttar Pradesh.
2. a
8) Consider the following statements with
respect to Stree Swabhiman Scheme • Though the elephant is the symbol of the BSP
and the lion is the symbol of the Forward Bloc,
1. It is an initiative for promoting women’s the EC in 1991 stopped allowing parties
menstrual health and hygiene. to use animals as symbols after
2. It comes under the Ministry of Women complaints from animal rights activists.
and Child Development. 3. a
Which of the statement(s) given above is/are • The Israeli spacecraft – Beresheet - the first
correct? privately funded mission to the Moon has
a. 1 only crashed on the lunar surface after the apparent
failure of its main engine.
b. 2 only
• The aim of the mission was to take pictures
c. Both 1 and 2 and conduct experiments.
d. Neither 1 nor 2
• Israel hoped to become the fourth country to
land a spacecraft on the Moon.
9) Consider the following pairs • Only government space agencies from the
former Soviet Union, the US and China have
Places in News – Country
made successful Moon landings.
1. Khartoum – Sudan
4. b
2. Quetta – Pakistan
• Uranium is a radioactive element.
3. Cape Canaveral – French Guiana
5. b

www.shankariasacademy.com | www.iasparliament.com
33

6. c 2. Rabindranath Tagore renounced his


knighthood in protest while Gandhi
• Fall Armyworm (FAW), or Spodoptera gave up the title of Kaiser-i-Hind,
frugiperda, is an insect that is native to tropical bestowed by the British for his work
and subtropical regions of the Americas. It is during the Boer War.
a nocturnal pest.
Select the correct code
• It prefers maize, but can feed on more than
80 additional species of crops, including rice, a. 1 only
sorghum, millet, sugarcane, vegetable crops b. 2 only
and cotton.
c. Both 1 & 2
• It cannot tolerate extreme temperatures
or excess rainfall. d. Neither 1 nor 2

• The Pest has spread to 10 Indian states in the


past eight months. But, it has not spread to 2) Which of the following statements is/are
the North-Western India as yet. correct about Periyar River?
7. d 1. Sabarimala temple is located on the
• Forward Policy – Auckland banks of the river.

• Policy of Masterly Inactivity – John 2. It is a perennial river.


Lawrence 3. It entirely flows through Kerala.
• Policy of Proud Reserve – Lytton Select the correct one
8. a a. 1 only
• Stree Swabhiman is an initiative for promoting b. 1 & 3 only
women‘s menstrual health and hygiene by
c. 2 & 3 only
the Ministry of Electronics and
Information Technology. d. All of the above
• Under this initiative, Common Service Centres
(CSC) will provide access to affordable, reliable
3) Which of the following statements is correct
and modern (eco-friendly) sanitary napkins to
about the project TRINITEA
adolescent girls and women in rural areas.
a. It is a project of NASA to find about the
9. b
evidence regarding the presence of Blackhole.
• Khartoum – Sudan b. It is a project of ESA to deviate the asteroids
• Quetta – Pakistan that collides with Earth.

• Cape Canaveral – U.S.A c. It aims at providing support to the Small Tea


Growers at the farm and market level
10. c
d. It is a triministerial project of Government of
India for the socio-economic development of
the girl child
13-04-2019
1) Consider the following states about Jallian
Wallah Bagh Tragedy 4) Which of the elements/compounds
constitute Solar e- waste?
1. Martyrs of this tragedy have been
included in the dictionary of Martyrs of 1. Uranium
Indian freedom struggle released by
2. Sulphur hexafouride
Indian Council of historical research
(ICHR). 3. Cadmium.

www.shankariasacademy.com | www.iasparliament.com
34

4. Selenium
Select the correct code 8) The digital single market strategy is
associated with
a. 1, 3 & 4 only
a. European Union
b. 2 & 3 only
b. OECD
c. 2, 3 & 4 only
c. BRICS
d. 1, 2 & 3 only
d. G 20

5) Match the following correctly


9) Consider the following statements about Sev
List 1 ( tribes) List 2 (State)
bhoj yojana
1. Deori - Assam
1. It is launched by Ministry of Food
2. Idu mishmi - Nagaland processing industries.
3. Reang - Mizoram 2. It is a scheme to provide financial
assistance on purchase of specific food
Select the correct code items by Charitable Religious
a. 1 only Institutions (CRIs) for free distribution
among people.
b. 1 & 2 only
Select the correct code
c. 3 only
a. 1 only
d. 2 & 3 only
b. 2 only
c. Both 1 & 2
6) Which of the following parameters is NOT
included in National Institutional Ranking d. Neither 1 nor 2
Framework?
a. Perception 10) Which of the following is NOT the
b. Outreach & Inclusivity members of Gulf cooperation council ?
c. Infrastructure 1. Yemen
d. Consulting and collaborative performance 2. Iran
3. Iraq
7) Which of the following statements is Select the correct code
INCORRECT about Mansabdari system of a. 1 only
Mughal empire?
b. 3 only
a. The king himself appointed the mansabdars,
and could enhance the mansab, lower it or c. 2 & 3 only
remove it.
d. All of the above
b. The mansabdari system was hereditary.
c. A record was kept of the description (‘huliya’)
of each horseman under a mansabdar and of Answers
branding (‘dag’) horses to prevent corruption 1. b
d. The recruitment of the mansabdars was made 2. d
on the recommendation of the Mirbakshi who
presented the person to the emperor 3. c

www.shankariasacademy.com | www.iasparliament.com
35

4. c Which of the statement(s) given above is/are


correct?
5. a
a) 1 only
6. c
b) 2 only
7. b
c) Both 1 and 2
8. a
d) Neither 1 nor 2
9. b
10. d
4) Which of the following was formed by Dr.
B.R. Ambedkar?
15-04-2019 1. Bahishkrit Hitakarini Sabha
1) AZURE Mission sometimes seen in the news 2. Independent Labour Party
recently is associated with which of the
following? 3. All India Scheduled Castes Federation
a) Space bugs 4. Peasants and Workers Party of India
b) Anti stalling Select the correct answer using the codes given
below:
c) Auroras
a) All except 4
d) Gravitational Lensing
b) All except 1 and 4
c) All except 2 and 4
2) Consider the following statements with
respect to Ebola d) All of the above
1. It is a viral disease.
2. It can also spread through air. 5) State of World Population 2019 report was
published by which of the following?
3. There is as yet no proven treatment
available for Ebola. a) UN Population Fund
Which of the statement(s) given above is/are b) World Health Organisation
correct? c) UN Commission on Population and
a) 1 only Development
b) 1 and 3 only d) Population Action International
c) 2 and 3 only
d) 1, 2 and 3 6) Umbrella Revolution sometimes seen in the
news, is associated with which of the
following?
3) Consider the following statements with a) China-Taiwan Conflict
respect to Grand Challenge Initiative (GCI) –
Cusp b) Venezuela Protests
1. It is a mission launched by China. c) Kurds war against ISIS
2. The primary goal of the program is to d) None of the above
discover and characterize planetary
systems and Earth-like planets around
nearby stars. 7) Which of the following is/are not the eight
core missions of National Action Plan on
Climate Change (NAPCC)?

www.shankariasacademy.com | www.iasparliament.com
36

1. National Water Mission Answers


2. National Mission for Sustaining the 1. c
Western Ghats Ecosystem
• AZURE measures ionospheric winds and
3. National Mission for Sustainable circulation to better understand auroral
Agriculture effects.
4. National Mission for promoting Electric • NASA has successfully launched the Auroral
Mobility Zone Upwelling Rocket
Select the correct answer using the codes given Experiment or AZURE mission on April 5
below: from the Andoya Space Center in Norway.
a) 4 only 2. b
b) 1 and 3 only • Ebola is a virus that can spread quickly and
can be fatal in up to 90% of cases.
c) 1 and 4 only
• Ebola virus disease is not an airborne
d) 2 and 4 only
infection.
• There is as yet no proven
8) Which of the following country’s military treatment available for EVD.
unit was recently designated as Foreign
3. d
Terrorist Organisation (FTO) by the United
States of America (USA)? • GCI CUSP is an International research
a) Iraq project between Norway, USA, Canada,
and Japan involving 12 sounding rockets
b) Venezuela launched from Andoya and Ny-Alesund
between 2018 and 2020.
c) Sudan
d) Iran • The recently launched Mission AZURE is one
of the nine missions being conducted as part of
the Grand Challenge Initiative (GCI) – Cusp (9
9) Anandamath, a semi-historical novel by Missions, 12 Rockets)
Bankim Chandra Chattopadhyay, is based on • It aims to explore the polar cusp—where
which of the following? Earth’s magnetic field lines bend down to meet
a) Poligar’s Revolt the poles and particles from space can enter
our atmosphere.
b) Moplah Uprisings
• It aims to investigate the physics of heating
c) Santhal Rebellion and charged particle precipitation in this
d) Sanyasi Revolt region called the geomagnetic cusp—one of the
few places on Earth with easy access to the
electrically charged solar wind that pervades
10) Bhadra Tiger Reserve is located in which of the solar system.
the following states? 4. a
a) Kerala • The Peasants and Workers Party of India was
b) Uttar Pradesh founded in 1947.

c) West Bengal • The party was founded in Maharashtra by


Tulsidas Jadhav, Keshavrao Jedhe and others.
d) Karnataka
5. a
• According to the newly released State of World
Population 2019 report of the United

www.shankariasacademy.com | www.iasparliament.com
37

Nations Population Fund (UNFPA), the 2. The BMC consists of a Chairperson, and
enrolment rate of girls is higher than that of six persons nominated by local bodies,
boys in Indian schools at both primary and including 1/3rd women and 18% SC/ST.
secondary levels.
Which of the statement(s) given above is/are
• Worldwide, the rate of enrolment of girls is correct?
lower than that of boys at primary school, and
a) 1 only
equal at secondary level.
b) 2 only
6. d
c) Both 1 and 2
• Umbrella Revolution of 2014: China –
Hongkong Conflict d) Neither 1 nor 2
• Over one lakh Hong Kongers blocked roads in
the city for three months to protest China’s 2) Consider the following statements with
denial of democratic reforms in the Special respect to Measles
Administrative Region of Hongkong.
1. It is a highly contagious disease caused
7. d by virus.
Eight core missions of NAPCC 2. The virus normally passed through
1. National Solar Mission direct contact and through the air.
2. National Mission for Enhanced Energy 3. It is a human disease but also known to
Efficiency occur in animals.
3. National Mission on Sustainable Habitat Which of the statement(s) given above is/are
correct?
4. National Water Mission
a) 1 only
5. National Mission for Sustaining the Himalayan
Ecosystem b) 3 only
6. National Mission for a Green India c) 1 and 2 only
7. National Mission for Sustainable Agriculture d) None
8. National Mission on Strategic Knowledge for
Climate Change 3) AI Everything (AIE) – The world’s foremost
8. d Artificial Intelligence Summit was planned to
be held in which of the following countries?
• The United States of America (USA) has
recently designated Iran's Revolutionary a) Japan
Guard (IRGC) as a Foreign Terrorist b) U.A.E
Organization (FTO).
c) China
9. d
d) Italy
10. d

4) Consider the following statements with


16-04-2019 respect to BBIN Motor Vehicle Agreement
(MVA)
1) Consider the following statements with
respect to Biodiversity Management 1. It was signed and ratified by all the
Committees (BMC) member countries.
1. Environment Protection Act (EPA) 1986 2. The Asian Development Bank (ADB) is
mandates all local bodies to setup BMC. providing technical, advisory, and

www.shankariasacademy.com | www.iasparliament.com
38

financial support to BBIN MVA 7) Migration and Development Brief was


initiative. recently released by which of the following
organisations?
Which of the statement(s) given above is/are
correct? a) International Organisation for Migration
a) 1 only b) International Monetary Fund
b) 2 only c) World Bank
c) Both 1 and 2 d) Migration International
d) Neither 1 nor 2
8) Consider the following statements with
respect to Black Hole Tragedy
5) Consider the following statements with
respect to Department of Investment and 1. It refers to the event of death of English
Public Asset Management (DIPAM) men after they were lodged in to a tiny
room called Black Hole in Hyderabad.
1. It assists in simplifying and
streamlining of IP processes, apart from 2. At that time, Shuja-ud-daula was the
undertaking steps for furthering IPR Nawab of Bengal.
awareness, commercialization and
Which of the statement(s) given above is/are
enforcement.
correct?
2. It works under the Ministry of Finance.
a) 1 only
Which of the statement(s) given above
b) 2 only
is/are incorrect?
c) Both 1 and 2
a) 1 only
d) Neither 1 nor 2
b) 2 only
c) Both 1 and 2
9) Consider the following statements with
d) Neither 1 nor 2
respect to International Finance Corporation
(IFC)
6) Consider the following statements with 1. It is a member of the International
respect to Nirbhay Monetary Fund (IMF).
1. It is India’s first indigenously designed 2. It is headquartered in London, U.K.
and developed Long Range supersonic
3. It focuses exclusively on the private
Cruise Missile.
sector in developing countries.
2. It has a range of 700 km to 1,000 km at
Which of the statement(s) given above is/are
altitudes as low as 100 m.
correct?
Which of the statement(s) given above is/are
a) 3 only
correct?
b) 1 and 3 only
a) 1 only
c) 2 and 3 only
b) 2 only
d) 1, 2 and 3
c) Both 1 and 2
d) Neither 1 nor 2
10) Consider the following pairs:
Places in News – Country
1. Notre Dame Cathedral– France

www.shankariasacademy.com | www.iasparliament.com
39

2. Darfur Region – Sudan • The landmark BBIN (Bangladesh, Bhutan,


India, and Nepal) Motor Vehicles Agreement
3. Haiti – Pacific Ocean
(MVA) was signed by Transport Ministers of
Which of the pair(s) given above is/are the BBIN in 2015.
correctly matched?
• It will allow seamless movement of passenger
a) 3 only and cargo vehicles among the four countries.
b) 1 and 2 only • Bangladesh, India and Nepal have agreed on
c) 1, 2 and 3 the operating procedures for passenger vehicle
movement in the sub-region under the MVA.
d) None
• Bhutan did not ratify it, demanding a cap
on vehicles entering its territory for some time.
Answers • The Asian Development Bank (ADB) is
providing technical, advisory, and financial
1. b
support to BBIN MVA initiative as part of its
• As per the National Biological Diversity assistance to South Asia Sub-regional
Act 2002, the local bodies constitute the Economic Cooperation (SASEC) program.
Biodiversity Management Committees (BMC).
5. a
• The BMC consists of a Chairperson, and six
• DIPAM has been mandated to advise the
persons nominated by local bodies, including
Union Government in the matters of financial
1/3rd women and 18% SC/ST.
restructuring of PSUs and also for attracting
• Kerala is the first state in India to have investment through capital markets.
completed the formation of Biodiversity
• It works under the Ministry of Finance.
Management Committees (BMC) in all local
self-government institutions. • CIPAM assists in simplifying and streamlining
of IP processes, apart from undertaking steps
• National Green Tribunal (NGT) has recently
for furthering IPR awareness,
directed the Ministry of Environment and
commercialization and enforcement.
Forests (MoEF) to submit a report on
constitution of Biodiversity Management • CIPAM works under the Ministry of Commerce
Committees (BMC) at the local level in every and Industries.
state within three months.
6. b
2. c
• Nirbhay is India’s first indigenously designed
• Measles is a human disease and is not known and developed Long Range Sub-Sonic Cruise
to occur in animals. Missile.
• World Health Organization (WHO) has • It can be deployed from multiple platforms.
recently raised alarm over a 300 percent
increase in the case of measles globally in the • The missile has the capability to loiter and
first quarter of 2019 compared with last year. cruise at 0.7 Mach, at altitudes as low as 100
m.
3. b
• The Mach number is defined as the ratio of the
• United Arab Emirates (UAE) is set to host speed of the aircraft to the speed of sound i.e
the world’s foremost Artificial Intelligence (AI) Mach 1 means the velocity is equal to the speed
Summit to empower global dialogue on the of sound.
future of government, business and society.
• When the velocity exceeds the speed of sound
• The inaugural 'AI Everything', AIE will take is called supersonic and if it is less than the
place at the Dubai World Trade Centre, speed of sound it is called subsonic.
between 30th of April and 1st of May.
7. c
4. b

www.shankariasacademy.com | www.iasparliament.com
40

8. d a) 1 only
• Siraj-ud-daula the Nawab of Bengal is b) 2 only
believed to have imprisoned 146 English
c) Both 1 and 2
persons who were lodged in the infamous
Black Hole of Calcutta due to which 123 of d) Neither 1 nor 2
them died of suffocation.
• However, historians either do not believe this 2) Consider the following statements with
story, or say that the number of victims must respect to Export Promotion Council for
have been much smaller. Handicrafts (EPCH)
9. a 1. It is the nodal export promotion body
• The International Finance Corporation (IFC) is for handicrafts in India.
an international financial institution that 2. It functions under the aegis of Ministry
offers investment, advisory, and asset of Commerce and Industries.
management services to encourage private
sector development in developing Which of the statement(s) given above is/are
countries. correct?
• It is a member of the World Bank a) 1 only
Group and is headquartered b) 2 only
in Washington, D.C., United States.
c) Both 1 and 2
10. b
d) Neither 1 nor 2
1. Notre Dame Cathedral– France
2. Darfur Region – Sudan
3) Consider the following statements with
• Haiti is in the western one-third of the island respect to Hridyam programme
of Hispaniola between the Caribbean Sea
and the North Atlantic Ocean. 1. It is a unique initiative by the
Government of Maharashtra.
• The UN Security Council has voted
unanimously to completely end its 2. It aims to support children with
peacekeeping operations in Haiti on October congenital heart diseases by offering
15 and establish a political mission to support free surgeries.
government efforts to promote order and Which of the statement(s) given above is/are
development in Latin America's poorest correct?
country.
a) 1 only
b) 2 only
17-04-2019 c) Both 1 and 2
1) Consider the following statements with d) Neither 1 nor 2
respect to Resilient Cities Asia-Pacific
Congress (RCAP) 2019
1. It will offer cities and regions from Asia- 4) Consider the following statements with
Pacific a variety of innovative solutions respect to Comptroller and Auditor General of
that build resilience to climate change at India
the sub-national level. 1. He/She is appointed by the president of
2. It was organized by the Ministry of India by a warrant under his hand and
Housing and Urban affairs. seal.
Which of the statement(s) given above is/are
correct?

www.shankariasacademy.com | www.iasparliament.com
41

2. He/She is an agent of the Parliament 2. Jainism did not condemn the Varna
and conducts audit of expenditure on system, as Buddhism did.
behalf of the Parliament.
Which of the statement(s) given above is/are
Which of the statement(s) given above is/are correct?
correct?
a) 1 only
a) 1 only
b) 2 only
b) 2 only
c) Both 1 and 2
c) Both 1 and 2
d) Neither 1 nor 2
d) Neither 1 nor 2

8) Consider the following statements with


5) Consider the following terms sometimes respect to Unnat Bharat Abhiyan (UBA)
seen in the news recently:
1. It aimed at providing rural India with
1. Sadr-e-Riyasat – President professional resource support from
institutes of higher education to address
2. Wazir-e-Azam – Prime Minister
the development challenges through
Which of the pair(s) given above is/are appropriate technologies.
correctly matched?
2. It was launched by the Ministry of
a) 1 only Human Resource Development.
b) 2 only Which of the statement(s) given above is/are
correct?
c) Both 1 and 2
a) 1 only
d) Neither 1 nor 2
b) 2 only
c) Both 1 and 2
6) Match the following
d) Neither 1 nor 2
Festivals – Regions
1. Vishu – a. Kerala
9) A Central Monitoring Committee to check
2. Rongali Bihu – b. Tamil Nadu river pollution was recently established by
3. Naba Barsha – c. Assam which of the following organisations?
4. Puthandu Pirappu – d. Bengal a) Central Pollution Control Board
Select the correct answer using the codes given b) Ministry of Environment and Forest
below: c) National Green Tribunal
a) 1-d; 2-c; 3-a; 4-b d) Ministry of Water Resources and River
b) 1-d; 2-a; 3-c; 4-b Development
c) 1-a; 2-c; 3-d; 4-b
d) 1-b; 2-d; 3-c; 4-a 10) Mojave Desert is located in which of the
following regions?
a) North America
7) Consider the following statements
b) South East Asia
1. Parshvanatha was the first and
Mahavira was the last of 24 Jain c) North Africa
Tirthankaras. d) Australia

www.shankariasacademy.com | www.iasparliament.com
42

• Rishabhanatha (Adinatha) was the first and


Mahavira was the last of 24 Jain Tirthankaras.
Answers
8. c
1. a
• Unnat Bharat Abhiyan aimed at providing
• The 4th Resilient Cities Asia-Pacific 2019 rural India with professional resource support
Congress is being organised from institutes of higher education to address
by International Council for Local the development challenges through
Environmental Initiatives (ICLEI) and appropriate technologies.
hosted by South Delhi Municipal Corporation
in New Delhi. • It aims to link the Higher Education
Institutions with at least (5) villages, to enable
2. a the institutions contribute to the economic and
• Export Promotion Council for Handicrafts social betterment of these village communities
(EPCH) was established under Companies using their knowledge base.
Act in the year 1986-87 and is a non-profit • IIT Delhi has been designated to function as
organisation. the National Coordinating Institute for this
• It is the nodal export promotion body for program.
handicrafts in India and plays an important 9. c
role with the main objective of boosting trade
in handicrafts and also projects India’s image • To check river pollution, National Green
in the global market as a reliable supplier. Tribunal has constituted a Central
Monitoring Committee to undertake a national
• It works under the aegis of Development initiative by way of preparation and
Commissioner (Handicrafts), Ministry of enforcement of a national plan to make river
Textiles. stretches pollution free.
• The 8th edition of HOME EXPO INDIA 2019 • It comprises of a senior representative of NITI
opened at India Expo Centre and Mart, Aayog, secretaries Ministry of Water
Greater Noida recently. Resources, Ministry of Urban Development,
• The three-day exhibition has been organized Ministry of Environment, Forest and Climate
by the Export Promotion Council for Change, Director General, National Mission
Handicrafts (EPCH). for Clean Ganga and chairman CPCB.

3. b • Chairman CPCB will be the nodal authority for


coordination.
• Hridyam is a unique initiative by
Government of Kerala to support children 10. a
with Congenital Heart Disease (CHD). • Mojave Desert, arid region of southeastern
• It aims to offer free surgeries for children with California and portions of Nevada, Arizona,
congenital heart diseases. and Utah, U.S.A (North America)

4. c
5. c 19-04-2019
6. c 1) Consider the following statements with
respect to Representation of the People Act,
7. b 1951
• Jainism did not condemn the Varna system, as 1. Under the Act, no person shall vote at
Buddhism did. any election if he/she is confined in a
• According to Mahavira, a person is born in a prison.
high or in a lower varna in consequence of the 2. It also prohibits a person to cast their
sins or the virtues acquired by him in the vote, if he/she is held under preventive
previous birth. detention .

www.shankariasacademy.com | www.iasparliament.com
43

Which of the statement(s) given above is/are 2. Recently, the participants of this
correct? exercise celebrated 25 years of
Comprehensive Strategic Partnership.
a) 1 only
Which of the statement(s) given above is/are
b) 2 only
correct?
c) Both 1 and 2
a) 1 only
d) Neither 1 nor 2
b) 2 only
c) Both 1 and 2
2) Consider the following statements with
d) Neither 1 nor 2
respect to Bubble Boy Disease
1. It is a disorder which causes babies to
be born with little to no immune 5) Khooni Vaisakhi sometimes seen in the
protection. news recently refers to
2. Bone marrow transplant is the only way a) Harvest festival in Punjab
to cure this disease.
b) Classical poem
Which of the statement(s) given above is/are
c) Ancient higher education institution
correct?
d) Religious procession
a) 1 only
b) 2 only
6) Consider the following statements with
c) Both 1 and 2
respect to Sea Vigil Exercise 2019
d) Neither 1 nor 2
1. It is the maiden national level Coastal
Disaster Response and Humanitarian
Exercise.
3) Consider the following statements with
respect to Tendered votes 2. It was conducted by Indian Navy in close
coordination with Indian Coast Guard.
1. A vote is said to be a tendered vote, if a
voter votes for more than one elections. Which of the statement(s) given above
is/are incorrect?
2. They can be casted on both Electronic
Voting Machines and ballot papers. a) 1 only
Which of the statement(s) given above is/are b) 2 only
correct?
c) Both 1 and 2
a) 1 only
d) Neither 1 nor 2
b) 2 only
c) Both 1 and 2
7) Which of the following are part
d) Neither 1 nor 2 of Triratnas in Jainism?
1. Right Speech
4) Consider the following statements with 2. Right Knowledge
respect to IN–VPN BILAT EX sometimes seen
3. Right Conduct
in the news recently
4. Right Determination
1. It is a bilateral maritime exercise held
between India and Vietnam. Select the correct answer using the codes given
below:

www.shankariasacademy.com | www.iasparliament.com
44

a) 3 only • The provisions however exempt a person


held under preventive detention from
b) 2 and 3 only
this rigour.
c) 1, 2 and 3 only
2. a
d) 2, 3 and 4 only
• Officially known as X-linked severe
combined immunodeficiency (X-
8) Bamako Convention is associated with SCID) is a disorder that causes babies to be
which of the following? born with little to no immune protection,
making them prone to developing life-
a) Poaching threatening infections.
b) Peat land Conservation • It is caused by a specific gene mutation.
c) Nuclear Weapon Free Zone • It affects 1 in 2,00,000 newborns, almost
d) Hazardous Wastes exclusively males.
• In one famous case, a boy with SCID, David
Vetter, lived in a sterile plastic “bubble” until
9) Oumuamua sometimes seen in the news his death at age 12 — hence the disease’s
recently refers to which of the following? nickname: Bubble Boy Disease.
a) Malicious Software • Until now, the best treatment for these difficult
b) Space debris cycle cases relied on using bone marrow transplants
from tissue-matched siblings to restore
c) Super Computer immunity.
d) Interstellar object • Because, previous gene therapy techniques
either did not fully restore immune function or
resulted in side effects as serious as leukemia.
10) Consider the following pairs
• Now eight babies with “bubble boy disease”
Passes – States have had it fixed by an advanced gene
1. Chang La – Sikkim therapy made from one of the immune
system’s worst enemies HIV, the virus that
2. Sela Pass – Jammu and Kashmir causes AIDS.
Which of the pairs(s) given above is/are 3. d
correctly matched?
• According to the Conduct of Elections Rules,
a) 1 only 1961, a voter is allowed to cast a ‘tendered vote’
b) 2 only where someone else representing to be a
particular voter has already cast that vote.
c) Both 1 and 2
• The presiding officer may allow the actual
d) Neither 1 nor 2 voter to vote, if the person is able to prove his
or her identity.

Answers • They would be provided a ballot paper to mark


symbol and it would be placed in a cover
1. a specially kept for the purpose.
• Section 62(5) of the Representation of People • A tendered ballot paper is the same as the
Act of 1951 mandates that “no person shall ballot paper displayed on the balloting unit,
vote at any election if he is confined in a except that it shall be endorsed on the back
prison, whether under a sentence of with the words, “Tendered Ballot Paper” either
imprisonment or transportation or otherwise, stamped by the returning officer or written by
or is in the lawful custody of the police”. the presiding officer at the time of issuing it.

www.shankariasacademy.com | www.iasparliament.com
45

4. a • Oumuamua is the first interstellar


object known to enter our solar system.
• IN–VPN BILAT EX 2019 is the second
edition of the bilateral maritime exercise, • Scientist have now proposed that before
held between Indian Navy and Vietnam ‘Oumuamua, there may have been another
Peoples’ Navy. recent interstellar visitor — a meteor from
another solar system may have actually hit
• Based on the need to tackle China both
Earth in 2014.
countries have been moving closer in their
strategic relationship. 10. d
• The first Memorandum of Understanding was • Chang La – Jammu and Kashmir
signed in 1994, a formal Defence Protocol in
2000 and Strategic Partnership in 2007. • Sela Pass – Arunachal Pradesh

• This has been upgraded to


the Comprehensive Strategic 20-04-2019
Partnership in 2016.
1) Consider the following statements with
• Vietnam has this sort of relationship with only respect to International Fleet Review 2019
Russia and China.
1. It is a parade of naval ships, aircraft and
5. b submarines organized by nations to
• A book containing English translation of promote goodwill, strengthen
the 100-year-old classic Punjabi cooperation and showcase their
poem about Jallianwala Bagh organizational capabilities.
massacre, 'Khooni Vaisakhi' has been 2. It was conducted by Indian Navy along
released recently in Abu Dhabi. the coast of Visakhapatnam in Andhra
6. a Pradesh.
Which of the statement(s) given above is/are
• Sea Vigil is the first coastal defence
correct?
Exercise conducted by the Indian Navy
and Indian Coast Guard. a) 1 only
• The exercise involves all the 13 coastal states b) 2 only
and Union Territories (UTs).
c) Both 1 and 2
• Conducted for the first time ever, it aims to d) Neither 1 nor 2
test the country’s preparedness to thwart any
attempt by anti-national elements to carry out
an attack on its territory or against its citizens
2) Consider the following statements with
by infiltrating through the sea route.
respect to Drugs Technical Advisory Board
7. b (DTAB)
• Right Faith, Right Knowledge and 1. It is the highest statutory decision
Right Conduct are known as Triratnas in making body on technical matters
Jainism. related to drugs in India.
8. d 2. It is a part of Central Drugs Standard
Control Organisation (CDSCO) in the
• The Bamako Convention is a treaty of African Ministry of Health and Family Welfare.
nations prohibiting the import into Africa
of any hazardous (including Which of the statement(s) given above is/are
radioactive) waste. correct?

• The convention came into force in 1998. a) 1 only

9. d b) 2 only

www.shankariasacademy.com | www.iasparliament.com
46

c) Both 1 and 2 7) Consider the following statements with


respect to Australia Group
d) Neither 1 nor 2
1. It promotes transparency and greater
responsibility in transfers of
3) World Press Freedom Index was released conventional arms and dual-use goods
annually by which of the following? and technologies.
a) United Nations 2. India and China are members of this
group.
b) Global Press Institute
Which of the statement(s) given above is/are
c) UNESCO correct?
d) Reporters without Borders a) 1 only
b) 2 only
4) India has recently rebuilt Chhyoiphel c) Both 1 and 2
Kundeling Monastery. It is located in which of
the following countries? d) Neither 1 nor 2
a) Bhutan
b) Sri Lanka 8) Index of Cancer Preparedness was prepared
by which of the following organisations?
c) Nepal
a) Economist Intelligence Unit
d) China
b) World Health Organisation
c) Centre for Disease Control and Prevention
5) Consider the following statements with
respect to Haemophilia d) Medicins Sans Frontier
1. It is a lifelong hereditary bleeding
disorder in which the blood does not
9) Varuna is a naval exercise between India
clot properly.
and _______
2. It is more prominent in males than in
a) France
females.
b) Russia
Which of the statement(s) given above is/are
correct? c) U.K
a) 1 only d) Canada
b) 2 only
c) Both 1 and 2 10) Consider the following statements with
respect to Mali
d) Neither 1 nor 2
1. It is a landlocked country located in
Western Africa.
6) Chenchu Tribal Community lives in which of
2. Equator passes through this country.
the following states?
Which of the statement(s) given above is/are
a) Kerala
correct?
b) Tamil Nadu
a) 1 only
c) Andhra Pradesh
b) 2 only
d) West Bengal
c) Both 1 and 2

www.shankariasacademy.com | www.iasparliament.com
47

d) Neither 1 nor 2 • This illness is a sex-linked recessive disorder.


Due to the sex-linkage of the disorder, there is
a greater prominence in males than in
Answers females.
1. a 6. c

• International Fleet Review (IFR) is a parade of • The Chenchus are Adivasi, a designated
naval ships, aircraft and submarines, and is Scheduled Tribe in the Indian states
organized by nations to promote goodwill, of Andhra Pradesh, Telangana,
strengthen cooperation and showcase their Karnataka and Odisha.
organizational capabilities. 7. d
• IFR also serves as an ideal platform for the • Australia Group seeks to ensure that exports
world’s navies to showcase their prowess and do not contribute to the development
indigenous ship designing and shipbuilding of chemical or biological weapons.
capabilities in a global/ international arena.
• It has 43 members including India.
• Indian Navy warships INS Kolkata and INS
Shakti will be participating in IFR 2019 to be • China is not a member 0f AG nor of
held in Qingdao, China. MTCR nor Wassenaar Arrangement.
2. c • Delegations representing the countries meet
every year in Paris.
• DTAB is the highest statutory decision-making
body on technical matters related to drugs in 8. a
India. It was constituted under the provisions
9. a
of the Drugs and Cosmetics Act, 1940.
10. a
• DTAB is part of the Central Drugs Standard
Control Organization (CDSCO) under the • Tropic of Cancer passes through Mali.
Ministry of Health and Family Welfare.
3. d
22-04-2019
• India has dropped two places on
a World Press Freedom Index to be 1) Project 15 Bravo sometimes seen in the news
ranked 140th out of 180 countries in the recently is associated with which of the
annual Reporters Without following?
Borders analysis released recently. a) Stealth Submarines
4. c b) Pest free bananas
5. c c) Solar Power plants
• Haemophilia is a hereditary bleeding disorder, d) None of the above
in which there is a partial or total lack of an
essential blood clotting factor.
• It is a lifelong disorder, that results in 2) Consider the following statements with
excessive bleeding, and many times respect to Indian Bull Frog
spontaneous bleeding, which, very often , is 1. It is native to Andaman and Nicobar
internal. Islands and invasive species remains a
great threat for its survival.
• Haemophilia A is the most common form,
referred to as classical haemophilia. 2. It is listed as critically endangered
under the IUCN Red list.
• It is the result of a deficiency in clotting factor
8, while haemophilia B (Christmas Disease) is Which of the statement(s) given above is/are
a deficiency in clotting factor 9. correct?

www.shankariasacademy.com | www.iasparliament.com
48

a) 1 only d) Neither 1 nor 2


b) 2 only
c) Both 1 and 2 6) Consider the following statements with
respect to Election Commission
d) Neither 1 nor 2
1. Election Commission is a multimember
body since its inception.
3) Consider the following statements with
2. It does not have the power to disqualify
respect to Chief Election Commissioner (CEC)
candidates who commit electoral
1. While the CEC can only be removed in malpractices.
the manner set out for a Supreme Court
3. It also does not have the power to
judge, the other two commissioners may
deregister any political party.
be removed on the recommendation of
the CEC. Which of the statement(s) given above is/are
correct?
2. The Election Commissioners are on a
par with the CEC and the latter is not a) 2 only
superior in standing with the other
b) 2 and 3 only
Commissioners.
c) 1, 2 and 3
Which of the statement(s) given above is/are
correct? d) None
a) 1 only
b) 2 only 7) Which of the following countries does not
share border with Black Sea?
c) Both 1 and 2
1. Turkey
d) Neither 1 nor 2
2. Georgia
3. Russia
4) Garia Puja is a festival celebrated in which
of the following regions? 4. Bulgaria
a) Tripura Select the correct answer using the codes given
below:
b) West Bengal
a) 4 only
c) Assam
b) 1 and 3 only
d) Odisha
c) 2 and 4 only
d) None
5) Consider the following statements with
respect to National Monuments Authority
(NMA)
8) Consider the following statements with
1. It works under the Ministry of Culture. respect to Trade Related Analysis of Fauna and
Flora in Commerce (TRAFFIC)
2. It does not have any provision to declare
monument as of national importance. 1. It is a bureau under United Nations
Environment Programme (UNEP).
Which of the statement(s) given above is/are
correct? 2. It works globally on trade in wild
animals and plants in the context of
a) 1 only
both biodiversity conservation and
b) 2 only sustainable development.
c) Both 1 and 2

www.shankariasacademy.com | www.iasparliament.com
49

Which of the statement(s) given above is/are • Indian bullfrogs introduced in the Andaman
correct? Islands are Invasive and eat native wildlife
including fish and lizards.
a) 1 only
b) 2 only • It is listed as Least Concern under the IUCN
Red List of Threatened Species.
c) Both 1 and 2
3. c
d) Neither 1 nor 2
4. a
5. c
9) Phen Wildlife Sanctuary is located in which
of the following states? • National Monuments Authority (NMA) under
the Ministry of Culture, Govt. of India has
a) Madhya Pradesh been setup as per provisions of The Ancient
b) Lakshadweep Monuments and Archaeological Sites
and Remains AMASR (Amendment and
c) Mizoram Validation) Act, 2010.
d) Maharashtra • Monuments are declared protected and
of national importance by
the Archaeological Survey of India
10) Gohain Uliuwa mela is a festival celebrated (ASI).
in which of the following states?
• National Monument Authority (NMA) does
a) Assam not have any provision to declare
b) Chhattisgarh monument/site as of national importance.

c) Manipur 6. b

d) Andaman & Nicobar Islands • The EC does not have the power to disqualify
candidates who commit electoral malpractices.
At best, it may direct the registration of a case.
Answers • The EC also does not have the power to
1. d deregister any political party.

• Indian Navy has recently launched • However, the Constitution empowers the EC to
advanced stealth guided missile decide whether a candidate has incurred
destroyer ship INS Imphal in water at disqualification by holding an office of profit
Mazagon docks in Mumbai, Maharashtra. under the appropriate government, or has
been declared an insolvent, or acquired the
• It is third in series of four stealth guided citizenship of a foreign state.
missile destroyer of Visakhapatnam class built
7. d
indigenously under Project-15 B (Bravo).
• It is the first warship to be christened
after city in the North-East.
• It is expected to be commissioned in 2023.
2. d
• The Indian bullfrog Hoplobatrachus
tigerinus (native to the Indian
subcontinent) has rapidly invaded the
Andaman Islands after it was introduced there
in the early 2000s.

www.shankariasacademy.com | www.iasparliament.com
50

8. b d) Neither 1 nor 2
• TRAFFIC is a leading non-governmental
organization working globally on trade in
3) Consider the following statements with
wild animals and plants in the context of both
respect to East Asian-Australasian Flyway
biodiversity conservation and sustainable
Partnership
development.
1. It is a part of United Nation
9. a
Environment Programme (UNEP) and
• Phen Wildlife Sanctuary is a popular was adopted in the list of the World
buffer zone of Kanha national park, Madhya Summit on Sustainable Development
Pradesh. (WSSD).

• It lies in Southern region of Kanha tiger 2. It is a voluntary initiative that aims to


reserve, close to Madhya Pradesh and protect migratory water birds, their
Chhattisgarh state borders. habitat and the livelihoods of people
dependent upon them.
• It was declared as a wildlife sanctuary in year
1983 by Government of Madhya Pradesh. Which of the statement(s) given above is/are
correct?
• The Fauna at this sanctuary mainly consists of
the Tiger, Leopard, Wild boar, Cheetal, a) 1 only
Sambar etc. b) 2 only
10. a c) Both 1 and 2
d) Neither 1 nor 2
23-04-2019
1) Extinction Rebellion movement sometimes 4) A Global Deal for Nature (GDN) sometimes
seen in the news recently is associated with seen in the news recently refers to which of the
which of the following? following?
a) Refugee migration a) It is a plan to save the diversity and abundance
of life on Earth
b) Threatened species
b) It is an intergovernmental initiative to protect
c) Climate Change forests and grasslands
d) None of the above c) It is an UN mission that aims to reduce carbon
emissions to zero by 2050
2) Consider the following statements with d) It is a plan by Small Island Developing States
respect to Pradhan Mantri Jeevan Jyoti Bima (SDIS) to combat global warming
Yojana (PMJJBY)
1. It aims to provide accidental insurance 5) Consider the following statements with
cover at a very affordable premium of respect to National Bamboo Mission (NBM)
Rs.12/year.
1. It aims to increase the area under
2. The scheme is being offered only by Life bamboo plantation in
Insurance Corporation of India (LIC). government's forest lands and also in
Which of the statement(s) given above is/are private lands to supplement farm
correct? income and contribute towards
resilience to climate change.
a) 1 only
2. It comes under the Ministry of
b) 2 only Agriculture and Farmers Welfare.
c) Both 1 and 2

www.shankariasacademy.com | www.iasparliament.com
51

Which of the statement(s) given above is/are 8) On April 21, 1526, a battle was fought
correct? between the invading forces of Babur and
Sultan Ibrahim Lodi – one of the earliest
a) 1 only
battles involving gunpowder firearms and field
b) 2 only artillery in the subcontinent. Babur won and
established the Mughal Empire in India. What
c) Both 1 and 2 is the name of this historical Battle?
d) Neither 1 nor 2 a) Battle of Panipat
b) Battle of Chanderi
6) Radio Detection and Ranging (RADAR) has c) Battle of Haldighati
applications in which the following areas?
d) Battle of Chausa
1. Safe landing of a plane
2. Locate rain and snow clouds
9) Gender Parity Index (GPI) is released by
3. Detect speeding cars which of the following organizations?
4. Anti-collision systems a) UNICEF
5. Create soil profiles b) World Economic Forum
Select the correct answer using the codes given c) UNESCO
below:
d) NITI Aayog
a) All except 1, 2 and 5
b) All except 2 and 5
10) Aegan Sea is located between which of the
c) All except 5 following countries?
d) All of the above a) Latvia and Sweden
b) Italy and Croatia
7) Consider the following statements with c) Turkey and Greece
respect to East Asian-Australasian Flyway
(EAAF) d) Italy and Tunisia
1. It is one of 9 major migratory routes
recognised globally.
Answers
2. It extends from Arctic Russia and North
America to the south Australian 1. c
boundaries.
• Extinction Rebellion is an international socio-
3. It includes most of the East Asian political movement which uses nonviolent
regions including Andaman and resistance to avert climate breakdown, halt
Nicobar Islands. biodiversity loss, and minimise the risk of
human extinction and ecological collapse.
Which of the statements given above are
correct? It has three core demands:
a) 1 and 2 only 1. For the government to "tell the truth about
climate change"
b) 1 and 3 only
2. To reduce carbon emissions to zero by 2025
c) 2 and 3 only
3. To create a citizens' assembly to oversee
d) 1, 2 and 3 progress.
2. d

www.shankariasacademy.com | www.iasparliament.com
52

• PMJJBY offers coverage for death due to • The main objective of National Bamboo
any reason and is available to people in the Mission (NBM) is to increase the area under
age group of 18 to 50 years (life cover up to age bamboo plantation in non-forestGovernment
55) having a savings bank account who gives lands and private lands to supplement farm
their consent to join and enable auto debit. income and contribute towards resilience to
climate change as well as availability of quality
• A life cover of Rs. 2 lakhs is available for a one raw material requirement of industries.
year period at a premium of Rs.330/- per
annum per member and is renewable every • The bamboo plantations will be promoted
year. predominantly in farmers’ fields, homesteads,
community lands, arable wastelands, and
• It is administered through LIC and other along irrigation canals, water bodies etc.
Indian private Life Insurance
companies. • The NBM will be a sub scheme of National
Mission on Sustainable Agriculture (NMSA)
3. b
under the umbrella scheme Krishonnati
• The East Asian-Australasian Flyway Yojana.
Partnership (EAAFP) is a network of partners 6. d
within the East Asian-Australasian Flyway
(EAAF). • Radio Detection and Ranging
(RADAR) works by sending radio signals
• It aims to protect migratory water birds, their towards an object and receiving the signals
habitat and the livelihoods of people that are reflected by the object.
dependent upon them.
• The reflected signals can be seen on a screen.
• The Flyway is one of 9 major migratory
routes recognised globally. Applications
• It was adopted in the list of the World Summit • In airports it is used to help planes land safely
on Sustainable Development as a Type II
• Weather forecasters use it to locate rain and
initiative which is informal and voluntary.
snow clouds
(It is not a part of UN)
• Ships’ captains use radar while travelling
• There are currently 37 Partners including 18
through fog and mist
countries, 6 intergovernmental agencies, 12
international non-governmental organisations • On land, policemen use radar to detect
(NGOs) and 1 international private enterprise. speeding cars
4. a • In geophysics, radars so called GPR-Systems,
• The Global Deal for Nature is a time-bound, are used to analyze the ground to create soil
science-based plan to save the diversity profiles
and abundance of life on Earth. 7. d
• Scientists has recently proposed this plan to • The East Asia/Australasia Flyway is one of 9
prevent another mass extinction event on the major migratory routes recognised
planet. globally.
The three overarching goals of the GDN are • The Flyway extends from Arctic Russia and
1. To protect biodiversity by conserving at least North America to the southern limits of
30 per cent of the Earth’s surface by 2030 Australia and New Zealand.

2. Mitigate climate change by conserving the • It encompasses large parts of East Asia, all of
Earth’s natural carbon storehouses Southeast Asia and includes eastern India and
the Andaman and Nicobar Islands.
3. Reduce major threats
• Researchers have recently discovered that for
5. b the first time, East Asian birds have made the
Great Nicobar Island as their stopover for a

www.shankariasacademy.com | www.iasparliament.com
53

few week rest before they can fly along the 2) Consider the following statements with
East Asian-Australasian Flyway (EAAF). respect to Atal Bimit Vyakti Kalyan Yojna
1. The scheme covers Insured Persons (IP)
covered under the Employees’ State
Insurance Act, 1948 and aims to provide
cash relief to unemployed insured
person.
2. It was launched by Ministry of Labour
and Employment.
Which of the statement(s) given above is/are
correct?
a) 1 only
b) 2 only
c) Both 1 and 2
d) Neither 1 nor 2

3) Consider the following statements with


respect to Missile Technology Control Regime
(MTCR)
1. It promotes transparency and greater
8. a responsibility in transfers of
conventional arms and dual-use goods
9. c and technologies.
10. c 2. India and China are the members of this
group.

24-04-2019 Which of the statement(s) given above is/are


correct?
1) Consider the following statements with
a) 1 only
respect to Kimberley Process Certification
System (KPCS) b) 2 only
1. It came in to effect through a United c) Both 1 and 2
Nations General Assembly Resolution.
d) Neither 1 nor 2
2. It is a binding agreement to remove
conflict diamonds from the global
supply chain. 4) Global Education Monitoring Report is
3. India is one of the founding members. published by which of the following
organisations?
Which of the statements given above are
correct? a) UNESCO

a) 1 and 2 only b) World Economic Forum

b) 2 and 3 only c) UNICEF

c) 1, 2 and 3 d) European Commission

d) None

www.shankariasacademy.com | www.iasparliament.com
54

5) Consider the following statements with b) Persian Gulf and Gulf of Oman
respect to Stars
c) Red Sea and Gulf of Oman
1. Stars are huge celestial bodies made
d) Persian Gulf and Gulf of Aden
mostly of hydrogen and helium that
produce light and heat from the
churning nuclear forges inside their
cores. 9) Mosquirix is a vaccine developed for which
of the following diseases?
2. The colour of a star depends on the
distance of the star from the observer. a) Malaria

Which of the statement(s) given above is/are b) Dengue


correct? c) Cholera
a) 1 only d) Japanese Encephalitis
b) 2 only
c) Both 1 and 2 10) Consider the following pairs
d) Neither 1 nor 2 Places in News – Country
1. Vladivostok – Georgia
6) Consider the following statements 2. Tunis – Tunisia
1. India is Iran’s largest oil buyer after 3. Algiers – Algeria
China.
Which of the pair(s) given above is/are
2. India is the world’s third-biggest oil correctly matched?
consumer, meets more than 80% of its
crude oil requirements through a) 2 only
imports. b) 3 only
Which of the statement(s) given above is/are c) 2 and 3 only
correct?
d) 1, 2 and 3
a) 1 only
b) 2 only
Answers
c) Both 1 and 2
d) Neither 1 nor 2
1. c
• Kimberley process is a joint initiative of
Government, International Diamond Industry
7) Significant Reduction Exceptions and Civil Society to stem the flow of Conflict
(SRE) sometimes seen in the news recently is Diamonds.
associated with which of the following?
• Conflict Diamonds are rough diamonds used
a) Waiver from US sanctions by rebel movements or their allies to finance
b) Kimberly Process conflict aimed at undermining legitimate
governments.
c) Cabotage laws
• It came into effect in 2003 through a UNGA
d) None of the above Resolution with representation from 81
countries including EU.

8) Strait of Hormuz is a strait connecting • India is one of the founding members.


which of the following? • It is a binding agreement to remove conflict
a) Red Sea and Gulf of Aden diamonds from the global supply chain.

www.shankariasacademy.com | www.iasparliament.com
55

2. c • It was granted to eight countries — India,


China, Japan, South Korea, Taiwan, Turkey,
• Employees' State Insurance
Italy and Greece for six months until May 2,
Corporation under Ministry of Labour
2019.
and Employment has recently rolled out
Atal Bimit Vyakti Kalyan Yojna. 8. b
• The scheme covers Insured Persons (IP) • The Strait of Hormuz is a strait between the
covered under the Employees’ State Insurance Persian Gulf and the Gulf of Oman.
Act, 1948.
• After the US said it would prevent five of Iran’s
• It aims to provide cash relief to unemployed biggest customers — including India — from
insured person. buying its oil, Tehran threatened to close the
Strait of Hormuz, a neck of water between its
• The relief will be payable in cash directly to southern coast and the northern tip of the
their Bank Account in case of unemployment sultanate of Oman, and the lane through which
and while they search for new engagement. a third of the world’s seaborne oil passes every
3. d day.
• It aims to limit the spread of ballistic missiles 9. a
and other unmanned delivery systems that • Mosquirix is the world’s first vaccine against
could be used for chemical, biological, and a parasitic disease, Malaria.
nuclear attacks.
• Malaria is caused by the infectious
• It has 35 members, which include most of the Plasmodium female Anopheles mosquitoes
world's key missile manufacturers, including deposit parasite sporozoites into the skin of a
India. human host.
• It seeks to restrict the exports of missiles and • Each year there are 430,000 malaria-related
related technologies of any type of weapon of deaths worldwide according to the World
mass destruction. Health Organization (WHO).
4. a
• Mosquirix vaccine has been recommended by
5. a WHO for pilot introduction in selected areas of
3 African countries- Ghana, Kenya and
• Stars come in lots of different colours, and Malawi.
their colour depends on the temperature of
the star. 10. c
6. c
• India, the world’s third-biggest oil consumer, 25-04-2019
meets more than 80% of its crude oil
1) Consider the following statements with
requirements and around 40% of its natural
respect to Echidnas
gas needs through imports.
1. It is a unique egg-laying mammal found
• India is Iran’s top oil buyer after China. In
predominantly in Indian Subcontinent.
2018-19, it imported 23.5 million tonnes from
Iran; in the previous year, almost 10% of its 2. Anti-microbial protein (AMP) identified
total 220.4 million tonnes of crude imports in the milk secreted by the Echidnas is
was from Iran. used to cure cancer.
7. a Which of the statement(s) given above is/are
correct?
• US granted waivers from its oil sanction from
Iran is also known as Significant Reduction a) 1 only
Exceptions (SRE). b) 2 only
c) Both 1 and 2

www.shankariasacademy.com | www.iasparliament.com
56

d) Neither 1 nor 2 4. Facilitate the growth of seed industry in


the country
Select the correct answer using the codes given
2) Consider the following statements with
below:
respect to Council of Scientific & Industrial
Research (CSIR) a) 4 only
1. It is an autonomous body under the b) 1, 2 and 3
Ministry of Science and Technology.
c) 1, 2, 3 and 4
2. The Shanti Swarup Bhatnagar Prize was
d) None
given annually by the CSIR.
Which of the statement(s) given above is/are
correct? 5) Recently, which of the following cities is
named as World Book Capital by UNESCO for
a) 1 only
the year 2019?
b) 2 only
a) Brazil
c) Both 1 and 2
b) Rome
d) Neither 1 nor 2
c) Sharjah
d) Sydney
3) Consider the following statements with
respect to International Criminal Police
Organization (Interpol) 6) Operation Epervier II sometimes seen in the
news recently is associated with which of the
1. It is an intergovernmental organization
following organisations?
facilitating international police
cooperation. a) Interpol
2. Its Headquarter is located in Lyon, Italy. b) World Wildlife Fund
3. It has 194 member countries, making it c) UNICEF
the world's largest police organization.
d) UNESCO
Which of the statement(s) given above
is/are incorrect?
a) 2 only 7) Beijing Declaration and Platform for
Action’, often seen in the news, is
b) 1 and 2 only
a) a strategy to tackle the regional terrorism, an
c) 2 and 3 only outcome of a meeting of the Shanghai
Cooperation Organization
d) None
b) a plan of action for sustainable economic
growth in the Asia-Pacific Region, an outcome
4) Which of the following is/are not the of the deliberations of the Asia-Pacific
objective(s) of Protection of Plant Varieties Economic Forum
and Farmers’ Rights (PPV&FR) Act, 2001?
c) an agenda for women’s empowerment, an
1. Protection of plant varieties outcome of a World Conference convened by
the United Nations
2. Protection of the rights of farmers and
plant breeders d) a strategy to combat wildlife trafficking, a
declaration of the East Asia Summit
3. Encourage the development of new
varieties of plants

www.shankariasacademy.com | www.iasparliament.com
57

8) Consider the following statements with Answers


respect to Kisan Urja Suraksha evam Utthaan
Mahabhiyan (KUSUM) Scheme 1. d

1. The Scheme aims to provide last mile • The Spiny anteater or Echidna, is a unique
electrical connectivity to all rural and egg-laying mammal found predominantly
urban households. in Australia and New Guinea.
2. The scheme required the consumer to • Their young hatch from eggs at a very early
mandatorily have a bank account linked stage of development and depend completely
with Aadhaar number for availing LPG on mother’s milk.
Subsidy.
• But the mammary glands of the echidnas are
3. It is implemented by Ministry of New devoid of nipples, forcing the young ones to
and Renewable Energy. lick milk from the mother’s body surface and
potentially making them vulnerable to
Which of the statement(s) given above is/are micro-organisms.
correct?
• However, the milk of the echidna has a protein
a) 3 only
that can puncture the cell membranes of
b) 1 and 2 only multiple bacterial species, thus destroying the
source of infection.
c) 2 and 3 only
• A research team at the CSIR-CCMB has been
d) None
able to show that the protein from echidna
milk is effective against mastitis-causing
bacteria (Mastitis, an infection of the
9) Global Democracy Index is published
mammary gland of lactating dairy
annually by which of the following
animals).
organisations?
• The protein promises to serve as an alternative
a) Economist Intelligence Unit
to antibiotics used on livestock.
b) Transparency International
2. c
c) World Economic Forum
3. a
d) UN Inter-agency Group
• Its Headquarter is located in Lyon, France.
4. d
10) Match the following:
Objectives of PPV & FR Act, 2001:
Mangoes – Region
1. To establish an effective system for
1. Alphonso – a. Karnataka the protection of plant varieties, the
2. Ishad – b. Gujarat rights of farmers and plant breeders
and to encourage the development of
3. Kesar – c. Maharashtra new varieties of plants.
Select the correct answer using the codes given 2. To recognize and protect the rights of farmers
below: in respect of their contributions made at any
time in conserving, improving and making
a) 1-a; 2-b; 3-c
available plant genetic resources for the
b) 1-b; 2-a; 3-c development of new plant varieties.
c) 1-c; 2-b; 3-a 3. To accelerate agricultural development
in the country, protect plant breeders’
d) 1-c; 2-a; 3-b
rights; stimulate investment for research and
development both in public & private sector
for the development new of plant varieties.

www.shankariasacademy.com | www.iasparliament.com
58

4. Facilitate the growth of seed industry in 2) Consider the following statements with
the country which will ensure the availability respect to Central Administrative Tribunal
of high quality seeds and planting material to (CAT)
the farmers.
1. The Tribunal is guided by the principles
5. c of natural justice and is not bound by
the procedure, prescribed by the Civil
6. a
Procedure Code.
• An INTERPOL Operation named Epervier 2. Appeals against the orders of the CAT
II have rescued 216 men, women and children could be made only in the Supreme
who were trafficked into forced labour and Court and not in the high courts.
prostitution in Benin and Nigeria.
Which of the statement(s) given above is/are
7. c correct?
8. a a) 1 only
• Kisan Urja Suraksha evam Utthaan b) 2 only
Mahabhiyan (KUSUM) is a farmer-oriented
solar power scheme that will allow setting up c) Both 1 and 2
grid-connected solar plants in rural areas and d) Neither 1 nor 2
off-grid solar pumps.
• It involves decentralized solar power
production of up to 28,250 Megawatt (Mw) 3) Consider the following statements with
over five years. respect to Centre for Cellular and Molecular
Biology (CCMB)
• It aims at boosting farmers’ income by
allowing them to sell additional power to the 1. Its main objective is to conduct research
grid through solar plants. in frontier and multi-disciplinary areas
of modern biology and to seek potential
• It is implemented by Ministry of New and applications of this work.
Renewable Energy.
2. It operates under the aegis of the
9. a Council of Scientific and Industrial
10. d Research (CSIR).

1. Alphonso – a. Maharashtra Which of the statement(s) given above is/are


correct?
2. Ishad – b. Karnataka
a) 1 only
3. Kesar – c. Gujarat
b) 2 only
c) Both 1 and 2
26-04-2019 d) Neither 1 nor 2
1) Kelchawa Festival is celebrated at the close
of the harvest season by ________tribes in the
state of ________ 4) Consider the following statements
a) Kuliya – Telangana 1. It is a combination of music, dance and
narrative.
b) Munda – Chhattisgarh
2. It is the only classical dance of India
c) Tiwa – Assam having links with Muslim culture.
d) Singpho – Arunachal Pradesh 3. It is the only form of classical dance
wedded to Hindustani or the North
Indian music.

www.shankariasacademy.com | www.iasparliament.com
59

Identify the classical dance that was mentioned 7) Consider the following statements with
above: respect to Yakshagana
a) Kathakali 1. It is a traditional theatre form of
Karnataka based on mythological
b) Kuchipudi
stories and Puranas.
c) Kathak
2. Though originated in Vijaynagar
d) Sattriya Empire, its most popular episodes are
about Mughal empires.
Which of the statement(s) given above is/are
5) Consider the following statements with correct?
respect to Global Forest Watch
a) 1 only
1. It is an online platform that provides
data and tools for monitoring forests. b) 2 only
2. It is an initiative of UN Environment c) Both 1 and 2
Programme in cooperation with CITES
d) Neither 1 nor 2
and World Wild life Fund.
Which of the statement(s) given above is/are
correct? 8) Anamalai Tiger Reserve (ATR) is located in
which of the following states?
a) 1 only
a) Kerala
b) 2 only
b) Tamil Nadu
c) Both 1 and 2
c) Karnataka
d) Neither 1 nor 2
d) Both A and B

6) Consider the following statements with


respect to Atal Bhoojal Yojana 9) World Wildlife Crime Report is published by
which of the following organisations?
1. The emphasis of the scheme will be on
recharge of ground water sources and a) World Wildlife Fund
revival of surface water bodies by
b) Interpol
involving people at the local level.
c) IUCN
2. The half of the fund will be supported by
a World Bank loan and the rest will be d) UN office on drugs and crime
funded by central government.
Which of the statement(s) given above is/are
correct? 10) Project Mausam is a project of which of the
following ministry?
a) 1 only
a) Ministry of Culture
b) 2 only
b) Ministry of Shipping
c) Both 1 and 2
c) Ministry of Commerce
d) Neither 1 nor 2
d) Ministry of Tourism

Answers
1. c
2. a

www.shankariasacademy.com | www.iasparliament.com
60

• The CAT is not bound by the procedure laid • Atal Bhoojal Yojana is aimed at efficient
down in the Civil Procedure Code of 1908. It is management of available water resources.
guided by the principles of natural justice.
• The emphasis of the scheme will be on
• Originally, appeals against the orders of the recharge of ground water sources and revival
CAT could be made only in the Supreme Court of surface water bodies by involving people at
and not in the high courts. the local level.
• However, in the Chandra Kumar case2 (1997), • It will focus on demand side management
the Supreme Court declared this restriction on (how to meet requirements by minimum use of
the jurisdiction of the high courts as water).
unconstitutional.
• The half of the fund will be supported by a
• Consequently, now it is not possible for an World Bank loan of 3000 crore and the rest
aggrieved public servant to approach the will be funded by central government.
Supreme Court directly against an order of the
CAT, without first going to the concerned high • It would initially be implemented with
court. community participation in Gujarat,
Maharashtra, Haryana, Karnataka, Rajasthan,
3. c Uttar Pradesh and Madhya Pradesh.
4. c • It focuses primarily on involvement of
communities and convergence with different
• The word Kathak has been derived from the
water scheme.
word Katha which means a story.
7. a
• Kathakars or story-tellers, are people who
narrate stories largely based on episodes from • Yakshagana, traditional theatre form of
the epics, myths and legends. Karnataka, is based on mythological stories
and Puranas.
• In both Hindu and Muslim courts, Kathak
became highly stylised and came to be • The most popular episodes are from
regarded as a sophisticated form of the Mahabharata and from Ramayana.
entertainment.
8. b
• Being the only classical dance of India having
9. d
links with Muslim culture, Kathak represents a
unique synthesis of Hindu and Muslim genius 10. a
in art.
• Further, Kathak is the only form of classical
dance wedded to Hindustani or the North 27-04-2019
Indian music. 1) Global Food Policy Report 2019 was
5. a published recently by which of the following
organisations?
• Global Forest Watch (GFW) is an online
platform that provides data and tools for a) Food and Agricultural Organisation
monitoring forests. b) International Food Policy Research Institute
• GFW is an initiative of the World Resources c) Indian Council of Medical Research
Institute (WRI), with partners including
Google, USAID, the University of Maryland d) Institute for Health Metrics and Evaluation
(UMD), Esri, Vizzuality and many other
academic, non-profit, public, and private
organizations.
6. c

www.shankariasacademy.com | www.iasparliament.com
61

2) Consider the following statements with a) 1 and 2 only


respect to National Human Rights
b) 2 only
Commission (NHRC)
c) 2 and 3 only
1. It is a constitutional body established
through 75th Constitutional d) 1, 2 and 3
Amendment Act, 1994.
2. The chairman and members of NHRC
are appointed by the President on the 5) Consider the following statements with
recommendations of Chief Justice of respect to Ek Bharat Shrestha Bharat
India. 1. It aims to actively enhance interaction
Which of the statement(s) given above is/are between people of diverse cultures
correct? living in different States and UTs, with
the objective of promoting greater
a) 1 only mutual understanding and bonding
amongst them.
b) 2 only
2. It comes under the Ministry of Human
c) Both 1 and 2
Resources and Development.
d) Neither 1 nor 2
Which of the statement(s) given above is/are
correct?
3) Consider the following statements with a) 1 only
respect to Sustainable Development Goals
b) 2 only
(SDGs)
c) Both 1 and 2
1. SDGs were born at the United Nations
Conference on Sustainable d) Neither 1 nor 2
Development in Rio de Janeiro in 2012.
2. SDG addresses multiple issues
surrounding poverty, sustainable 6) How is the Sustainable Development Goals
development, inequality and climate (SDGs) different from the Millennium
change. Development Goals (MDGs)?

Which of the statement(s) given above is/are 1. SDGs are universal and apply to all
correct? countries, whereas the MDGs were
intended for action in developing
a) 1 only countries only.
b) 2 only 2. MDGs are not legally binding whereas
SDGs are legally binding.
c) Both 1 and 2
Which of the statement(s) given above is/are
d) Neither 1 nor 2
correct?
a) 1 only
4) Consider the following statements with
b) 2 only
respect to Arms Trade Treaty, 2013
c) Both 1 and 2
1. It is a legally binding treaty.
d) Neither 1 nor 2
2. It seeks to regulate the flow of weapons
into conflict zones.
3. India signed and ratified the treaty in 7) Consider the following statements with
201 respect to Human Rights Council
Which of the statement(s) given above is/are 1. It is an inter-governmental body within
correct? the UN system responsible for

www.shankariasacademy.com | www.iasparliament.com
62

promoting and protecting human rights a) 3 only


around the world.
b) 1 and 3 only
2. The Council is made up of 47 member
c) 2 and 3 only
states which are elected by the UN
General Assembly (UNGA). d) 1, 2 and 3
Which of the statement(s) given above is/are
correct?
Answers
a) 1 only
1. b
b) 2 only
2. d
c) Both 1 and 2
• The National Human Rights Commission is
d) Neither 1 nor 2
a statutory (and not a Constitutional)
body.
8) Global Talent Competitiveness Index • It was established in 1993 under a legislation
2019 was published recently by which of the enacted by the Parliament, namely, the
following organisations? Protection of Human Rights Act, 1931.
a) INSEAD • This Act was amended in 2006.
b) World Bank • The chairman and members are appointed by
c) U.S. Chambers of commerce the president on the recommendations of a six-
member committee consisting of the Prime
d) World Economic Forum Minister as its head, the Speaker of the Lok
Sabha, the Deputy Chairman of the Rajya
Sabha, leaders of the Opposition in both the
9) Malaria Elimination Research Alliance Houses of Parliament and the Central home
India (MERA India) Program was recently minister.
launched by which of the following
organisations? 3. c

a) Indian Medical Association 4. a

b) National Institute of Malaria Research • India is not a signatory to the Arms Trade
Treaty, 2013 of UN.
c) National Centre for Disease Control
• The Arms Trade Treaty (ATT) is the
d) Indian Council of Medical Research first legally-binding instrument ever
negotiated in the United Nations to establish
common standards for the international
10) Consider the following statements with transfer of conventional weapons.
respect to Shigellosis
• US President has recently announced that his
1. It is a contagious disease caused by country will not abide by the UN ATT.
Shigella Virus.
• U.S.A signed the treaty but not yet ratified the
2. Humans and a few primates are the only treaty.
reservoir of Shigella.
5. c
3. Shigella are spread by direct contact
with an infected person, or by eating • Ek Bharat Shrestha Bharat was
contaminated food or drinking announced on the occasion of the 140th birth
contaminated water. anniversary of Sardar Vallabhbhai Patel.
Which of the statement(s) given above is/are • It aims to actively enhance interaction between
correct? people of diverse cultures living in different
States and UTs, with the objective of

www.shankariasacademy.com | www.iasparliament.com
63

promoting greater mutual understanding and 10. c


bonding amongst them.
• Shigellosis is a diarrheal disease caused by a
• Each year, every State/UT would be paired group of bacteria called Shigella.
with another State/UT in India for reciprocal
interaction between the people. • It is bloody diarrhoea, that is, stool would be
accompanied with blood.
• Rashtriya Ekta Shivir (National Integration
Camp) was organized to implement this • It mainly affects the large intestine and it is
program. recognised as a disease mainly affecting Asian
and African nations.
6. a
• Shigella are spread by direct contact with an
• The Sustainable Development Goals (SDGs) infected person, or by eating contaminated
are universal and apply to all countries, food or drinking contaminated water.
whereas the MDGs were intended for action in
developing countries only. • Flies may also transmit the organism.

• The Sustainable Development Goals (SDGs) • Humans and a few primates are the only
are not legally binding. reservoir of Shigella.

7. c
• The Human Rights Council is an inter- 29-04-2019
governmental body within the United 1) Suresh Mathur Committee was recently
Nations system. constituted for which of the following
• It is responsible for strengthening the purposes?
promotion and protection of human rights a) To lay down standards for metro rail systems
around the globe and for addressing situations
of human rights violations and make b) To review the regulatory framework on micro
recommendations on them. insurance

• It meets at the UN Office at Geneva. c) To recommend measures to boost domestic


output of oil and gas agencies
• The Council is made up of 47 United
Nations Member States which are elected d) Inter Country removal and retention of
by the UN General Assembly. Children through Hague Convention

• The Human Rights Council replaced the


former United Nations Commission on Human 2) Consider the following statements with
Rights. respect to Belt and Road Initiative (BRI)
8. a 1. It is a giant land and maritime
connectivity project, driven to revive the
• India ranks 80 in Global Talent
Ancient Silk Road.
Competitiveness Index 2019.
2. Italy is the first country to formally join
• It is released by INSEAD business school in the Belt and Road Initiative (BRI).
partnership with Tata Communications and
Adecco Group. 3. India for the first time participated in
the second edition of Belt and Road
9. d Forum (BRF) conference that was
• The Indian Council of Medical concluded recently.
Research has launched the 'Malaria Which of the statement(s) given above is/are
Elimination Research Alliance (MERA) India' - correct?
a conglomeration of partners working on
malaria control - in order to prioritise, plan a) 1 only
and scale up research to eliminate the disease b) 1 and 2 only
from India by 2030.

www.shankariasacademy.com | www.iasparliament.com
64

c) 1 and 3 only
d) 1, 2 and 3 6) Manjira Wildlife Sanctuary is located in
which of the following states?
a) Telangana
3) Kafala system sometimes seen in the news
recently was followed by which of the following b) Gujarat
countries?
c) Assam
a) Qatar
d) Rajasthan
b) Australia
c) Indonesia
7) Priyadarshini is sometimes seen in the news
d) Maldives recently. It is
a) Super Computer
4) Consider the following statements with b) Coast Guard Vessel
respect to INS Vikramaditya
c) Women Welfare Scheme
1. It is a Hermes class aircraft carrier
d) Anti Ballistic Missile
commissioned by the Royal Navy, UK.
2. It plays a major role in Indo-Pak War of
1971. 8) Arrange the following from South to North
Which of the statement(s) given above is/are 1. Thal Ghat
correct?
2. Bhor Ghat
a) 1 only
3. Pal Ghat
b) 2 only
Select the correct answer using the codes given
c) Both 1 and 2 below:
d) Neither 1 nor 2 a) 1-2-3
b) 3-2-1
5) Consider the following statements with c) 3-1-2
respect to Safe Cities Programme
d) 2-3-1
1. The programme implements women
safety initiatives in every state’s capital
across India. 9) Consider the following statements with
2. The projects are funded under Nirbhaya respect to Indian Council of Medical Research
Fund scheme. (ICMR)

3. The Centre and the States share the 1. It is responsible for appraisal of
expenses for the scheme in the ratio of activities involving large scale use of
60:40. hazardous microorganisms and
recombinants in research and industrial
Which of the statement(s) given above is/are production from the environmental
correct? angle.
a) 2 only 2. It is funded by the Ministry of Science
and Technology.
b) 3 only
Which of the statement(s) given above is/are
c) 2 and 3 only
correct?
d) 1, 2 and 3
a) 1 only

www.shankariasacademy.com | www.iasparliament.com
65

b) 2 only • Qatar is set to abolish its controversial exit


visa system for all foreign workers by the end
c) Both 1 and 2
of 2019 the UN’s International Labour
d) Neither 1 nor 2 Organization has recently said.
• The Kafala (Sponsorship) System emerged in
10) With reference to National Crisis the 1950’s to regulate the relationship between
Management Committee (NCMC), consider the employers and migrant workers in many
following statements: countries in West Asia.

1. It is headed by the Prime Minister of • It remains the routine practice in the Gulf
India. Cooperation Council (GCC) countries of
Bahrain, Kuwait, Oman, Qatar, Saudi Arabia
2. It can give directions to any ministry for and the United Arab Emirates (UAE), and also
specific action needed for meeting the in the Arab states of Jordan and Lebanon.
crisis situation.
• Under the Kafala system a migrant
Which of the statement(s) given above is/are worker’s immigration status is legally
correct? bound to an individual employer or
a) 1 only sponsor (kafeel) for their contract period.

b) 2 only • The migrant worker cannot enter the country,


transfer employment nor leave the country for
c) Both 1 and 2 any reason without first obtaining explicit
d) Neither 1 nor 2 written permission from the kafeel.
4. d

Answers • INS Vikramaditya is a Kiev class aircraft


carrier which was commissioned by Russian
1. b Navy in 1987 under the name Baku.
• The Insurance Regulatory and • It was later renamed as Admiral Gorshkov and
Development Authority of India later offered to India in 2004.
(IRDAI) has recently set up a committee to
review the regulatory framework on • It is the biggest and heaviest ship to be
microinsurance and recommend operated by the Indian Navy.
measures to increase the demand for 5. c
such products.
• In order to instil sense of security in women in
• Suresh Mathur as the chairman, the 13- metro cities, Government has identified eight
member panel has been tasked with suggesting cities for implementation of Safe City
product designs with customer-friendly projects in first phase at a cost of Rs.2,919
underwriting, including easy premium crore.
payment methods and simple claims
settlement procedures. • The cities are Ahmedabad, Bengaluru,
Chennai, Delhi, Hyderabad, Kolkata, Lucknow
2. a & Mumbai.
• Italy is the first G-7 country to formally join • The projects are funded under Nirbhaya Fund
the Belt and Road Initiative (BRI). scheme.
• India has decided to skip the second • The Centre and States share the expenses for
Belt and Road Forum held in Beijing in the scheme (60:40).
opposition to the BRI’s China-Pakistan
Economic Corridor (CPEC). • The projects are being implemented by
Ministry of Home Affairs in consultation with
3. a Ministry of Women and Child Development,
Ministry of Urban Development, Ministry of

www.shankariasacademy.com | www.iasparliament.com
66

Electronic and Information Technology, • The other members of the committee include
respective Municipal & Police Commissioners the Secretary to the Prime Minister,
of the cities and civil society organizations. Secretaries of Ministry of Home Affairs,
Defence, Research and Analysis Wing and
6. a
Agriculture and Cooperation 24 along with
7. b Director, Intelligence Bureau and officer of
Cabinet Secretariat.
• The First of the Class of 05 Fast Patrol Vessels
(FPVs), named “ICGS- Priyadarshini” built • The NCMC can give directions to any
by Garden Reach Shipbuilders and Engineers Ministry/Department/Organization for
Limited (GRSE), Kolkata was commissioned specific action needed for meeting the crisis
recently at the Kakinada Deep Water Port. situation.
• The remaining four ships are in advanced • The National Crisis Management Committee
stages of construction. (NCMC) met recently under the chairmanship
of Cabinet Secretary and took stock of the
• The FPV is a medium range surface vessel situation arising out of the Cyclonic Storm
capable of operating in the maritime zones of ‘Fani’.
India.
• These powerful, fuel-efficient platforms are
designed to perform multipurpose operations 30-04-2019
like patrolling, anti-smuggling, anti-poaching
and rescue operations. 1) Consider the following statements with
respect to Goldman Environmental Prize
• The entire design of these FPVs has been
1. It has been awarded annually by the UN
developed in-house by GRSE as per
Environment.
requirements specified by Indian Coast Guard.
2. It honours grassroots environmental
8. b
heroes from the world’s six inhabited
9. d continental regions.
• The Indian Council of Medical Research Which of the statement(s) given above is/are
(ICMR), New Delhi, the apex body in India for correct?
the formulation, coordination and promotion
a) 1 only
of biomedical research, is one of the
oldest medical research bodies in the b) 2 only
world.
c) Both 1 and 2
• The ICMR is funded by the Government of d) Neither 1 nor 2
India through the Department of Health
Research, Ministry of Health & Family
Welfare.
2) “Despatch with No Destination
• The Genetic Engineering Appraisal Alternate” method is sometimes seen in the
Committee (GEAC) functions under the news recently. It is associated with which of the
Ministry of Environment, Forest and Climate following?
Change (MoEF&CC) is responsible for a) Tariff barrier
appraisal of activities involving large scale use
of hazardous microorganisms and b) Fuel conservation Initiative
recombinants in research and industrial
c) Import Policy
production from the environmental angle.
d) Expedition exercise
10. b
• Under the chairmanship of the cabinet
Secretary the NCMC has been constituted in
the cabinet secretariat.

www.shankariasacademy.com | www.iasparliament.com
67

3) Consider the following statements with c) Both 1 and 2


respect to National Clean Air Programme
d) Neither 1 nor 2
(NCAP)
1. It aims to reduce particulate matter
(PM) pollution by 20%-30% in at least 6) Consider the following statements with
102 cities by 2024. respect to Airports Authority of India (AAI)
2. It is envisioned as a five-year action plan 1. Airports Authority of India (AAI) was
with 2020 as the first year. constituted by an Act of Parliament.
3. There would be a review after every 10 2. Its function includes design,
years. development, operation and
maintenance of international and
Which of the statement(s) given above is/are
domestic airports and civil enclaves.
correct?
Which of the statement(s) given above is/are
a) 1 only
correct?
b) 1 and 2 only
a) 1 only
c) 2 and 3 only
b) 2 only
d) 1, 2 and 3
c) Both 1 and 2
d) Neither 1 nor 2
4) With reference to the Ikshvaku dynasty,
consider the following statements
7) “Phanigiri” is a Buddhist site located in
1. Ikshvaku dynasty was founded after the
which of the following states?
fall of Vijayanagar empire.
a) Bihar
2. Though they followed Buddhism, they
also patronized Vedic ritualism. b) Maharashtra
Which of the statement(s) given above is/are c) Telangana
correct?
d) Uttar Pradesh
a) 1 only
b) 2 only
8) Consider the following statements with
c) Both 1 and 2 respect to Shanghai Cooperation Organisation
(SCO)
d) Neither 1 nor 2
1. It is a permanent intergovernmental
international organisation comprises of
5) Consider the following statements with eight member states.
respect to International Crops Research
2. It was preceded by the Shanghai Five
Institute for the Semi-Arid Tropics (ICRISAT)
mechanism.
1. It is an international non-profit
3. Its official languages are Russian,
organization that undertakes scientific
Hindi, Arabic and Chinese.
research for development.
Which of the statement(s) given above is/are
2. It is headquartered in Hyderabad,
correct?
India.
a) 2 only
Which of the statement(s) given above is/are
correct? b) 1 and 2 only
a) 1 only c) 1, 2 and 3
b) 2 only d) None

www.shankariasacademy.com | www.iasparliament.com
68

• Regulations require every flight to file


an alternative city or airport enroute
9) Consider the following pairs
where the aircraft can land and carries
Defence Systems – Country adequate fuel to cover such emergency.
1. Spike-LR – Israel • This means that the aircraft carries additional
2. Igla-S – Russia fuel which is a dead weight if there is no
diversion from the intended destination.
Which of the pair(s) given above is/are
correctly matched? • The heavier the aircraft, the more fuel it
consumes while flying.
a) 1 only
• Air India’s Delhi-Hyderabad flight has recently
b) 2 only created a new record in Indian aviation history
c) Both 1 and 2 when its Boeing 787 Dreamliner landed in
Hyderabad using a new method
d) Neither 1 nor 2 called ‘Dispatch with No Destination
Alternate’.

10) Consider the following statements with • (i.e.) without provision for additional fuel
respect to Pharmaceuticals Export Promotion which was hitherto necessary to carry in case
Council of India (Pharmexcil) of flight diversion.

1. The main objective of Pharmexcil is to 3. a


promote the export of Drugs and • The Union Environment Ministry has recently
Pharmaceuticals including ayurvedic constituted a committee to implement the
and homeopathic medicines. National Clean Air Programme (NCAP), which
2. It was setup by the Ministry of aims to reduce particulate matter (PM)
Commerce & Industry. pollution by 20%-30% in at least 102 cities by
2024.
Which of the statement(s) given above is/are
correct? • The NCAP is envisaged as a scheme to provide
the States and the Centre with a framework to
a) 1 only combat air pollution.
b) 2 only • The NCAP is envisioned as a five-year action
c) Both 1 and 2 plan with 2019 as the first year. There would
be a review every five years.
d) Neither 1 nor 2
4. d
• The fall of the Satavahana Empire left
Answers Andhra in political chaos.
1. b • Local rulers carved out small kingdoms for
• The Goldman Environmental Prize also known themselves. Most important among these
as Green Nobel Prize was awarded annually small dynasties was the Ikshvaku.
by Goldman Environmental • The first king of the Ikshvaku dynasty
Foundation. is Vasisthiputra Sri Chamtamula.
• It honours grassroots environmental heroes • Nagarjuna Konda was their capital and
from the world’s six inhabited continental they patronized Buddhism, though they
regions: Africa, Asia, Europe, Islands & followed the Vedic ritualism.
Island Nations, North America, and South &
Central America. • Sanskrit mostly replaced Prakrit as the
language of the inscriptions.
2. b
5. c

www.shankariasacademy.com | www.iasparliament.com
69

• The International Crops Research Institute for • It was preceded by the Shanghai Five
the Semi-Arid Tropics (ICRISAT) is a non- Mechanism.
profit, non-political organizationthat
conducts agricultural research for • Currently, the SCO comprises of eight
development in the dry lands of Asia and sub- member states, namely the Republic
Saharan Africa. of India, the Republic of Kazakhstan, the
People's Republic of China, the Kyrgyz
• ICRISAT and its partners help empower these Republic, the Islamic Republic of Pakistan, the
poor people to overcome poverty, hunger and a Russian Federation, the Republic of Tajikistan,
degraded environment through better and the Republic of Uzbekistan.
agriculture.
• The SCO's official languages are Russian and
• ICRISAT is headquartered in Hyderabad, Chinese.
Telangana State, in India, with two regional
9. c
hubs (Nairobi, Kenya and Bamako, Mali).
6. c • Spike-LR – Israel

• Airports Authority of India (AAI) was • Igla-S – Russia


constituted by an Act of Parliament: Airports 10. c
Authority of India Act, 1994 and came
into being on 1st April 1995 by merging • The main Objective of Pharmexcil is to
erstwhile National Airports Authority and support, protect, maintain, increase and
International Airports Authority of India. promote the export of Drugs and
Pharmaceuticals inter alia including
7. c intermediates, herbal, ayurvedic, unani and
8. b homeopathic medicines, biotech and biological
products, diagnostics, surgicals, nutraceuticals,
• The Shanghai Cooperation Organisation (SCO) pharma industry related services –
is a permanent intergovernmental collaborative research, contract
international organisation. manufacturing, providing base for clinical
trials and consultancy.

www.shankariasacademy.com | www.iasparliament.com

Das könnte Ihnen auch gefallen